Module 5 - Cash, Receivables, Inventory, and PP&E

Ace your homework & exams now with Quizwiz!

After being held for 40 days, a 120-day, 12% interest-bearing note receivable was discounted at a bank at 15%. The proceeds received from the bank equal Maturity value less the discount at 12%. Maturity value less the discount at 15%. Face value less the discount at 12%. Face value less the discount at 15%.

Maturity value less the discount at 15%. The bank charges its discount (its fee) on the maturity value, which is the face value of the note plus 12% interest for 120 days. The bank charges 15% on this amount for the 80 remaining days in the note term. Thus, the proceeds equal the maturity value less its fee.

Milton Co. pledged some of its accounts receivable to Good Neighbor Financing Corporation in return for a loan. Which of the following statements is correct? Good Neighbor Financing cannot take title to the receivables if Milton does not repay the loan. Title can only be taken if the receivables are factored. Good Neighbor Financing will assume the responsibility of collecting the receivables. Milton will retain control of the receivables. Good Neighbor Financing will take title to the receivables, and will return title to Milton after the loan is paid.

Milton will retain control of the receivables. You Answered Correctly! In a pledge arrangement, the title remains with the originator, in this case with Milton Co.

A corporation issued debt to purchase 10 acres of land for development purposes. Expenditures related to this purchase are as follows: Description Amount Purchase price $1,000,000 Real estate taxes in arrears 15,000 Debt issuance costs 2,000 Attorney fee—title search on land 5,000 The company should record its acquisition of the land in its financial statements at a value of $1,000,000. $1,015,000. $1,020,000. $1,022,000.

$1,020,000. Correct! The costs capitalized to the land are all costs to get the land ready for use (development). Those costs are: the cost of the land, the real estate taxes in arrears, and the attorney fee for the title search.

The following information relates to Jay Co.'s accounts receivable for 20x4: Accounts receivable, 1/1/x4$ 650,000 Credit sales for 20x4 2,700,000 Sales returns for 20x4 75,000 Accounts written off during 20x4 40,000 Collections from customers during 20x4 2,150,000 Estimated future sales returns at 12/31/x4 50,000 Estimated uncollectible accounts at 12/31/x4 110,000 What amount should Jay report for accounts receivable, before allowances for sales returns and uncollectible accounts, on December 31, 20x4?

$1,085,000 AR 1/1 + Credit sales − Sales returns − Write-offs − Collections = AR 12/31 $650,000 + $2,700,000 − $75,000 − $40,000 − $2,150,000 = $1,085,000

Data for a firm using the Average LCM (conventional retail inventory method) retail inventory method is as follows: Cost Retail Beginning inventory $ 300 $ 467 Net purchases 1,200 2,000 Net additional markups 100 Net markdowns(300) Sales$1,700 Compute cost of goods sold. $1,169 $1,160 $1,177 $1,122

$1,169 Ending inventory at retail = $567 (= $467 + $2,000 + $100-$300-$1,700). The cost-to-retail ratio includes both the beginning inventory amounts, purchases and net markups. C/R = $300 + $1,200/($467 + $2,000 + $100) = .5843. Ending inventory at cost = $567(.5843) = $331. Cost of goods sold = $300 + $1,200 - $331 = $1,169.

Cook Co. had the following balances on December 31, year 4: Cash in checking account$350,000 Cash in money market account250,000 U.S. Treasury bill, purchased 12/1/Y4, maturing 2/28/Y5 800,000 U.S. Treasury bond, purchased 3/1/Y4, maturing 2/28/Y5 500,000 Cook's policy is to treat as cash equivalents all highly liquid investments with a maturity of three months or less when purchased. What amount should Cook report as cash and cash equivalents in its December 31, year 4, balance sheet?

$1,400,000 Correct! The first three items in the list are included in cash and cash equivalents. If no restrictions apply, cash in checking accounts ($350,000) is always included in cash. Per ASC Topic 305, cash equivalents are short-term, highly liquid investments that are readily convertible into cash and have maturities of three months or less from the date of purchase by the entity. Common examples are Treasury bills, commercial paper, and money market funds. In this case, the cash equivalents are the money market account ($250,000) and the Treasury bill ($800,000). Therefore, total cash and cash equivalents is $1,400,000 ($350,000 + $250,000 + $800,000). The maturity of the Treasury bond was at least 12 months (3/1/Y4 to 2/28/Y5) from the date of purchase; therefore, it should not be reported in cash and cash equivalents. The reason for the three-month rule is to minimize price fluctuations due to interest rate changes. A security with a fluctuating price is not "equivalent" to cash. One year is too long a time to expect interest rates to remain stable.

Immediately after a note payable was signed, its present value was $30,000. This note and $20,000 cash were used to acquire a used plant asset at the beginning of the current year. The interest rate implied in the note is 6%. Total interest payments due on the note over its term amount to $4,000. The term exceeds one year. No payments on the note are due during the current year. What amount of interest expense is recognized for the first year (current year) on this note, and what amount is capitalized to the plant asset account? Interest Expense Capitalized Amount $1,800 $50,000 $3,000 $50,000 $4,000 $30,000 $0 $50,000

$1,800 $50,000 The interest expense recognized for the first year is .06($30,000) = $1,800. Although no interest is paid, interest is accrued, increasing the carrying value of the note. The asset is capitalized at $50,000, the sum of cash down payment and present value of the note. The interest over the note term is not capitalized because it does not assist in the process of placing the asset into its intended condition and location.

On June 1, 2005, Yola Corp. loaned Dale $500,000 on a 12% note, payable in five annual installments of $100,000 beginning January 2, 2006. In connection with this loan, Dale was required to deposit $5,000 in a noninterest-bearing escrow account. The amount held in escrow is to be returned to Dale after all principal and interest payments have been made. Interest on the note is payable on the first day of each month beginning July 1, 2005. Dale made timely payments through November 1, 2005. On January 2, 2006, Yola received payment of the first principal installment plus all interest due. On December 31, 2005, Yola's interest receivable on the loan to Dale should be $0 $5,000 $10,000 $15,000

$10,000 Because the last interest payment was made on November 1, the interest for November and December is unpaid as of December 31, 2005. Therefore, two months of interest is receivable, as of December 31, 2005, for a total receivable of $10,000 = (2/12)(12%)($500,000). No principal payments have yet been made as of this date.

Orr Co. prepared an aging of its accounts receivable at December 31, year 5 and determined that the net realizable value of the receivables was $250,000. Additional information is available as follows: Allowance for uncollectible accounts at 1/1/x5 - credit balance $ 28,000 Accounts written off as uncollectible during year 523,000 Accounts receivable at 12/31/x5 270,000 Uncollectible accounts recovery during year 55,000 For the year ended December 31, year 5, Orr's uncollectible accounts expense would be

$10,000 Beginning allowance balance + uncollectible accounts expense − write-offs + recoveries = ending allowance balance $28,000 + uncollectible accounts expense − $23,000 + $5,000 = ($270,000 − $250,000) Uncollectible accounts expense = $10,000 Under the aging method, the ending allowance balance equals the difference between gross accounts receivable ($270,000) and net realizable value of accounts receivable ($250,000). Write-offs decrease the allowance balance, and uncollectible accounts expense increases the allowance. Recoveries also increase the allowance because the amount by which the allowance was decreased when the account was written off is reinstated on recovery.

On October 20, 2005, Grimm Co. consigned 40 freezers to Holden Co. for sale at $1,000 each and paid $800 in transportation costs. On December 30, 2005, Holden reported the sale of 10 freezers and remitted $8,500. The remittance was net of the agreed 15% commission. What amount should Grimm recognize as consignment sales revenue for 2005? $7,700 $8,500 $9,800 $10,000

$10,000 Consignment sales revenue is the revenue recognized on consignment sales. In this case, total consignment revenue is 10 × $1,000 = $10,000. The commission and transportation costs are expenses that reduce earnings on consignment revenues, but they do not affect total revenues to be recognized.

On December 31, 1999, Key Co. received two $10,000 noninterest-bearing notes from customers in exchange for services rendered. The note from Alpha Co., which is due in nine months, was made under customary trade terms, but the note from Omega Co., which is due in two years, was not. The market interest rate for both notes at the date of issuance is 8%. The present value of $1 due in nine months at 8% is .944. The present value of $1 due in two years at 8% is .857. At what amounts should these two notes receivable be reported in Key's December 31, 1999, balance sheet? Alpha Omega $ 9,440 $ 8,570 $10,000 $8,570 $ 9,440 $10,000 $10,000 $10,000

$10,000 $8,570 The note from Alpha Co. is a short-term asset. It is reported at the face value of $10,000. The note from Omega is discounted as a single sum for two time periods at 8% to be reported at $10,000X.857=$8,570.

On January 2, Year 1, Lem Corp. bought machinery under a contract that required a down payment of $10,000, plus 24 monthly payments of $5,000 each, for total cash payments of $130,000. The cash-equivalent price of the machinery was $110,000. The machinery has an estimated useful life of 10 years and estimated salvage value of $5,000. Lem uses straight-line depreciation. In its Year 1 Income Statement, what amount should Lem report as depreciation for this machinery? $10,500 $11,000 $12,500 $13,000

$10,500 The capitalized cost of the equipment is $110,000, not the total of the cash payments to be made. The latter amount includes interest. Thus, annual depreciation is $10,500:($110,000-$5,000)/10.

On January 1, 20X5, Dix Co. replaced its old boiler. The following information was available on that date: Carrying amount of old boiler $ 8,000 Fair value of old boiler 2,000 Purchase and installation price of new boiler 100,000 The old boiler was sold for $2,000. What amount should Dix capitalize as the cost of the new boiler? $92,000 $94,000 $98,000 $100,000

$100,000 The disposal of the old boiler and purchase of the new boiler are separate transactions. The loss on disposal has no effect on the capitalized cost of the new boiler, which is recorded at its $100,000 purchase cost.

Vore Corp. bought equipment on January 2, 20X4 for $200,000. This equipment had an estimated useful life of five years and a salvage value of $20,000. Depreciation was computed by the 150% declining balance method. The accumulated depreciation balance at December 31, 20X5 should be: $102,000 $98,000 $91,800 $72,000

$102,000 Depreciation in 20X4 = $200,000(1.50/5) =$ 60,000 Depreciation in 20X5 = ($200,000-$60,000)(1.50/5) =42,000 Accumulated depreciation balance at the end of 20X5$ 102,000 The declining balance class of depreciation method does not deduct salvage value when computing depreciation although care must be taken not to depreciate the asset below salvage value. Also, the rate of depreciation applied to book value is the percentage of the method (150% in this case) divided by the useful life of the asset. Double declining balance, for example, is 200%/n or 2/n where n = useful life.

Inge Co. determined that the net value of its accounts receivable on December 31, 2005, based on an aging of the receivables, was $325,000. Additional information is as follows: Allowance for uncollectible accounts - 1/1/05$ 30,000 Uncollectible accounts written off during 2005 18,000 Uncollectible accounts recovered during 200 52,000 Accounts receivable at 12/31/05 350,000 For 2005, what would be Inge's uncollectible accounts expense?

$11,000 This question requires a determination of the pre-adjustment balance in the allowance account, and the ending balance. The difference between these two amounts is the increase in the account needed, which is also the amount recognized as bad debt expense. The aging method first determines the required ending balance in the allowance account, and then places the amount needed to increase the account to this required balance into the allowance account. The pre-adjustment allowance balance =Beginning balance - Write-offs + Recoveries =$30,000 − $18,000 + $2,000 = $14,000 The ending allowance balance =$350,000 ending gross AR − $325,000 ending net value of AR = $25,000 Therefore, bad debt expense is the amount needed to bring the allowance balance up to the ending balance of $25,000. The increase needed is $11,000 ($25,000 − $14,000).

A company with a June 30 fiscal year-end entered into a $3,000,000 construction project on April 1 to be completed on September 30. The cumulative construction-in-progress balances at April 30, May 31, and June 30 were $500,000, $800,000, and $1,500,000, respectively. The interest rate on company debt used to finance the construction project was 5% from April 1 through June 30 and 6% from July 1 through September 30. Assuming that the asset is placed into service on October 1, what amount of interest should be capitalized to the project on June 30? $11,666 $18,750 $75,000 $90,000

$11,666 CORRECT! Interest is capitalized on the project's average expenditures times the interest during that period. Key here is to use the 5% annual interest over the three months (April, May, and June). Average expenditures:500,000+800,000+1,500,000/3=933,333 Interest rate for 3-month period:.05 × 3/12 = *.0125 Capitalized interest: 11,666

Main Co. began its manufacturing business last year. Main uses the dollar-value LIFO method to determine the value of its inventory. Main's inventory was valued at $100,000 at the end of last year, and, using current costs, $132,000 at the end of the current year. The prices for Main's inventory during the current year were 20% higher than last year's prices. What amount should Main report as inventory on its balance sheet at the end of the current year? $110,000 $112,000 $122,000 $132,000

$112,000 Correct. Dollar-value LIFO requires that you adjust end-of-year inventory to prior-year costs ($132,000 ÷ 1.20 = $110,000). Prior year inventory of $100,000 compared to the current year inventory of $110,000, which means that inventory increased by $10,000. The increase needs to be adjusted for price increases of 20% and added to the beginning-of-year inventory. Ending dollar-value LIFO inventory is $100,000 + 12,000 ($10,000 × 1.20) = $112,000.

The following information pertained to Azur Co. for the year: Purchases $102,800 Purchase discounts 10,280 Freight-in 15,420 Freight-out 5,140 Beginning inventory 30,840 Ending inventory 20,560 What amount should Azur report as cost of goods sold for the year? $102,800 The following information pertained to Azur Co. for the year: Purchases$102,800Purchase discounts10,280Freight-in15,420Freight-out5,140Beginning inventory30,840Ending inventory20,560 What amount should Azur report as cost of goods sold for the year? $102,800 $118,220 $123,360 $128,500

$118,220 Cost of goods sold is determined (in a periodic inventory system) as: Beginning Inventory+ Net Purchases= Goods Available for Sale- Ending Inventory= Cost of Goods Sold Net Purchases includes any purchase discounts (or allowances) and other cost of getting the goods in place and condition for resale, including freight-in. Freight-out (to customers) is a selling cost. Therefore, Azur Co.'s cost of goods sold would be: Beginning Inventory $ 30,840 + Net Purchases $107,940 = Goods Available for Sale $138,780 - Ending inventory 20,560 = Cost of Goods Sold $118,220 Note: Net purchases calculated as: Purchases $102,800 - Purchase Discounts 10,280 + Freight In 15,420 = Net Purchases $107,940

The following information pertains to Grey Co. on December 31, year 3: Checkbook balance $12,000 Bank statement balance 16,000 Check drawn on Grey's account, payable to a vendor, dated and recorded 12/31/Y3 but not mailed until 1/10/Y4 1,800 On Grey's December 31, year 3 balance sheet, what amount should be reported as cash?

$13,800 The correct cash balance is the balance per the checkbook ($12,000) plus the $1,800 check written to the vendor, for a total of $13,800

On October 1, 20X4, Shaw Corp. purchased a machine for $126,000 that was placed in service on November 30, 20X4. Shaw incurred additional costs for this machine, as follows: Shipping$ 3,000Installation4,000Testing5,000 In Shaw's December 31, 20X4 Balance Sheet, the machine's cost should be reported as: $126,000 $129,000 $133,000 $138,000

$138,000 Every cost listed is a cost necessary to place the asset into its intended condition and location. This is the general rule for capitalizing costs to plant assets. Thus, all four costs are capitalized to the machine yielding a final capitalized value of $138,000 ($126,000 + $3,000 + $4,000 + $5,000).

The following information pertains to Park Co. on December 31: Bank statement balance $10,000 Checkbook balance 14,000 Deposit in transit 5,000 Outstanding checks 1,000 In Park's December 31 balance sheet, cash should be reported as

$14,000 As a check, take the ending balance per the bank + deposits in transit − outstanding checks; $10,000 + $5,000 − $1,000 = $14,000.

West Retailers purchased merchandise with a list price of $20,000, subject to trade discounts of 20% and 10%, with no cash discounts allowable. West should record the cost of this merchandise as $14,000 $14,400 $15,600 $20,000

$14,400 This is a chain discount and the correct recorded cost is $20,000(1 - .20)(1- .10) = $14,400. Each successive discount in a chain discount is applied to the previous net amount.

Marr Co. had the following sales and accounts receivable balances, prior to any adjustments at year end: Credit sales $10,000,000 Accounts receivable 3,000,000 Allowance for uncollectible accounts (debit balance) 50,000 Marr uses 3% of accounts receivable to determine its allowance for uncollectible accounts at year end. By what amount should Marr adjust its allowance for uncollectible accounts at year end? $0 $40,000 $90,000 $140,000

$140,000 The amount of the adjustment to get the $50,000 debit balance to a $90,000 (3% × $3,000,000) credit balance is $140,000.

On January 2, year 5, Ames Corp. signed an eight-year lease for office space. Ames has the option to renew the lease for an additional four-year period on or before January 2, year 13. During January year 5, Ames incurred the following costs: $120,000 for general improvements to the leased premises with an estimated useful life of 10 years. $50,000 for office furniture and equipment with an estimated useful life of 10 years. At December 31, year 5, Ames' intentions as to the exercise of the renewal option are uncertain. A full year's amortization of leasehold improvements is taken for calendar year 1. In Ames' December 31, year 5, Balance Sheet, accumulated amortization should be: $10,000 $15,000 $17,000 $21,250

$15,000 The appropriate amortization period for the leasehold improvements is eight years because renewal is uncertain. $120,000 ÷ 8 = $15,000. This is the amount in accumulated amortization because the property has been leased only one year. The office furniture and equipment are not included in leasehold improvements.

The following information pertains to Deal Corp.'s 2004 cost of goods sold: Inventory, 12/31/03 $ 90,000 2004 purchases 124,000 2004 write-off of obsolete inventory 34,000 Inventory, 12/31/04 30,000 The inventory written off became obsolete due to an unexpected and unusual technological advance by a competitor. In its 2004 income statement, what amount should Deal report as cost of goods sold? $218,000 $184,000 $150,000 $124,000

$150,000 Beginning inventory$ 90,000 Plus purchases 124,000 Less write-off(34,000) Less ending inventory(30,000) Equals cost of goods sold$150,000 The write-off cannot be counted in cost of goods sold because it is a decrease in inventory not associated with sales.

Oak Co., a newly formed corporation, incurred the following expenditures related to land and building: County assessment for sewer lines $ 2,500 Title search fees 625 Cash paid for land with a building to be demolished 135,000 Excavation for construction of basement 21,000 Removal of old building $21,000 less salvage of $5,000 16,000 At what amount should Oak record the land? $138,125 $153,500 $154,125 $175,625

$154,125 You Answered Correctly! The amounts necessary to get the land ready for its intended purpose attach themselves as a part of the total cost of the land. This would be the: $2,500+625+135,000+16,000=$154,125

Merry Co. purchased a machine costing $125,000 for its manufacturing operations and paid shipping costs of $20,000. Merry spent an additional $10,000 testing and preparing the machine for use. What amount should Merry record as the cost of the machine? $155,000 $145,000 $135,000 $125,000

$155,000 All three costs are included for a total of $155,000. Both the shipping and testing costs are necessary to place the asset into its intended location and condition for use. This is the criterion for capitalizing costs on acquisition of plant assets.

During a reporting period, a computer manufacturing company used raw materials of $50,000, had direct labor costs of $75,000, and factory overhead of $30,000. Other expenses were for advertising of $5,000, staff salaries of $10,000, and bad debt of $3,000. The company did not have a beginning balance in any inventory account. All goods manufactured during the period were sold during the period. What amount was the company's cost of goods sold during the reporting period? $155,000 $160,000 $170,000 $173,000

$155,000 Correct! Since there is no beginning or ending inventory, the cost of goods sold equals the cost of materials used ($50,000), the direct labor costs ($75,000), and the ($30,000) or $155,000. The other costs, advertising, staff salaries, and bad debt expense, are not part of cost of goods sold.

On June 18, 20X5, Dell Printing Co. incurred the following costs for one of its printing presses: Purchase of collating and stapling attachment $84,000 Installation of attachment 36,000 Replacement parts for overhaul of press 26,000 Labor and overhead in connection with overhaul 14,000 The overhaul resulted in a significant increase in production. Neither the attachment nor the overhaul increased the estimated useful life of the press. What amount of the above costs should be capitalized? $0 $84,000 $120,000 $160,000

$160,000 All four costs should be capitalized because they result in an increase in the productivity of the asset. Costs that increase EITHER the life OR productivity are capitalized. Either type of increase results in enhanced asset values. $160,000 is the sum of the four costs listed.

Information for a firm using the dollar value (DV) LIFO retail method follows. The cost to retail (C/R) is provided along with price level indices. The data reflects the use of the method through year one. Retail Retail DV LIFO Layer Base Index Current C/R Cost Base $200 1.00 $200 .40 $80 year one 80 1.10 88 .34 $30 For year two, ending inventory at retail (by count) totaled $450. The ending price-level index for the year was 1.15. The cost-to-retail ratio was .42. What is the ending inventory for financial reporting purposes for this firm? $164 $54 $189 $177

$164 The DV LIFO retail process applies the DV LIFO method to retail dollars, and then deflates the retail layer added, now reflecting current prices, to cost, using the cost-to-retail ratio. The calculations are: Ending inventory, retail, at base = $450(1.00/1.15) = $391 Increase in retail, current = $111(1.15/1.00) = $128 Increase in cost = $128(.42) = $54 Ending inventory at cost = ($80 + $30) + $54 = $164

Hall Co.'s allowance for uncollectible accounts had a credit balance of $24,000 on December 31, 2003. During 2004, Hall wrote off uncollectible accounts of $96,000. The aging of accounts receivable indicated that a $100,000 allowance for doubtful accounts was required on December 31, 2004. What amount of uncollectible accounts expense should Hall report for 2004?

$172,000 Beginning allowance balance $24,000 Less write-off (96,000) Equals pre-adjustment allowance balance(72,000)(debit) The allowance balance normally is a credit. For the ending balance in the account after adjustment to be $100,000 credit, the account must be increased $172,000 by recognizing uncollectible accounts expense. Under the aging method, uncollectible accounts expense equals the amount required to increase the allowance balance to the indicated total based on the aging of receivables.

In preparing its August 31 bank reconciliation, Apex Corp. has the following information available: Balance per bank statement, August 31 $18,050 Deposit in transit, August 313,250 Return of customer's check for insufficient funds, August 31600 Outstanding checks, August 312,750 Bank service charges for August 100 On August 31 Apex's correct cash balance is

$18,550. Balance per bank statement $18,050 Plus deposit in transit 3,250 Less outstanding checks (2,750) Equals ending cash balance $18,550

On April 1, 20X4, Kew Co. purchased new machinery for $300,000. The machinery has an estimated useful life of five years, and depreciation is computed by the sum-of-the-years'-digits method. The accumulated depreciation on this machinery at March 31, 20X6 should be: $192,000 $180,000 $120,000 $100,000

$180,000 $180,000, the correct answer, equals $300,000[(5 + 4)/(5 + 4 + 3 + 2 + 1)]. Two full years of depreciation have been recorded, and the SYD method uses the number of years left at the beginning of each year as the numerator of the fraction used in depreciation. At the beginning of the first and second years, five and four years of the asset's life remained, respectively. The denominator is the sum of the digits up to the asset's useful life (5).

Union Corp. uses the conventional retail method of inventory valuation. The following information is available: Cost Retail Beginning inventory $12,000 $ 30,000 Purchases 60,000 110,000 Net additional markups 10,000 Net markdowns 20,000 Sales revenue 90,000 If the lower of cost or market rule is used, what would be the estimated cost of the ending inventory? $24,000 $20,800 $20,000 $19,200

$19,200 The cost to retail ratio using the retail inventory method is: [$12,000 + $60,000/($30,000 + $110,000 + $10,000)] = .48. Ending inventory at retail is $30,000 + $110,000 + $10,000 − $20,000 − $90,000 = $40,000. Ending inventory at cost, therefore, is .48($40,000) = $19,200.

On July 1, year 5, Lee Co. sold goods in exchange for a $200,000, 8-month, noninterest-bearing note receivable. At the time of the sale, the note's market rate of interest was 12%. What amount did Lee receive when it discounted the note at 10% on September 1, year 5? $194,000 $193,800 $190,000 $188,000

$190,000 Six months remain in the note term at the date of discounting. Maturity value of note:$200,000 Less discount: $200,000(.10)(6/12)(10,000) Equals proceeds on note$190,000

Steven Corporation began operations in year 1. For the year ended December 31, year 1, Steven made available the following information: Total merchandise purchases for the year $350,000 Merchandise inventory at December 31, year 1 70,000 Collections from customers 200,000 All merchandise was marked to sell at 40% above cost. Assuming that all sales are on a credit basis and all receivables are collectible, what should be the balance in accounts receivable at December 31, year 1?

$192,000 This answer is correct. The first step is to determine sales for year 1. The cost of goods sold in year 1 is $280,000 ($350,000 purchases less $70,000 ending inventory). Note that beginning inventory is zero because the company began operations in year 1. Since all merchandise was marked to sell at 40% above cost, year 1 credit sales are $392,000 ($280,000 cost of goods sold × 140%). The second step is to determine the ending balance in accounts receivable.

On December 1, 2005, Tigg Mortgage Co. gave Pod Corp. a $200,000, 12% loan. Pod received proceeds of $194,000 after the deduction of a $6,000 nonrefundable loan origination fee. Principal and interest are due in 60 monthly installments of $4,450, beginning January 1, 2006. The repayments yield an effective interest rate of 12% at a present value of $200,000 and 13.4% at a present value of $194,000. What amount of accrued interest receivable should Tigg include in its December 31, 2005 balance sheet? $4,450 $2,166 $2,000 $0

$2,000 The term "accrued interest receivable" refers to the cash amount of interest due. The cash amount of interest due is based on the contractual interest rate and face value. The loan origination fee is a way of increasing the effective interest but it does not affect the cash interest component. The $2,000 accrued interest = (.12)(1/12)($200,000).

Paxton Co. signed contracts for the purchase of raw materials to be executed the following year at a firm price of $5 million. The market price of the materials dropped to $3 million on December 31. What amount should Paxton record as an estimated liability on purchase commitments as of December 31? $5,000,000 $3,000,000 $2,000,000 $0

$2,000,000 Correct! A liability is recorded only for the portion of the decline in market price of the raw materials relative to the price in the purchase commitment— a $2 million loss ($5 million − $3 million) because the loss is probable and estimable. The loss is probable because the entity has a firm contract to buy the materials for $5 million and the materials are worth only $3 million. So instead of deferring the loss, GAAP requires that the entity recognized the loss in the period that it occurred.

Adam Co. reported sales revenue of $2,300,000 in its income statement for the year ended December 31, year 5. Additional information was as follows: 12/31/x4 12/31/x5 Accounts receivable $500,000 $650,000 Allowance for uncollectible accounts (30,000) (55,000) Uncollectible accounts totaling $10,000 were written off during year 5. Under the cash basis of accounting, Adam would have reported year 5 sales of

$2,140,000 Under the cash basis of accounting, sales equals cash collected from customers. An equation or T account may be used to determine this amount: AR, beginning+Sales−Write-offs−customer collections =AR, ending$500,000$2,300,000$10,000?=$650,000 Solving for the unknown (?) amount, customer collections equals $2,140,000. This is the amount collected from customers, and is the amount that would be reported as sales under the cash basis method of accounting.

A company manufactured 1,000 units of product during the year and sold 800 units. Costs incurred during the current year are as follows: Direct materials and direct labor:$7,000Indirect materials and indirect labor:2,000Insurance on manufacturing equipment:3,000Advertising:1,000 What amount should be reported as inventory in the company's year-end balance sheet? $1,400 $1,800 $2,600 $2,400

$2,400 Correct! The $12 cost per unit of inventory is determined by adding the cost of the inventory for the year of $12,000 (7,000 + 2,000 + 3,000) divided by the number of units produced (1,000). Since 800 units were sold, 200 units remain in ending inventory. The total value of ending inventory is $2,400 ($12 cost per unit × 200 units). The advertising cost is a selling cost, not an inventory cost.

Marr Corp. reported rental revenue of $2,210,000 in its cash basis federal income tax return for the year ended November 30, 2004. Additional information is as follows: Rents receivable − November 30, 2004 $1,060,000 Rents receivable − November 30, 2003 800,000 Uncollectible rents written off during the fiscal year 30,000 Under the accrual basis, Marr should report rental revenue of

$2,500,000 The cash basis revenue in the tax return is the amount of rent collected for tax purposes. beg. rent receivable+accrual revenue−collections−write-offs=end. rent receivable$800,000+accrual revenue−$2,210,000−$30,000=$1,060,000 accrual revenue=$2,500,000

In year 6, Spirit, Inc. determined that the 12-year estimated useful life of a machine purchased for $48,000 in January year 1 should be extended by three years. The machine is being depreciated using the straight-line method and has no salvage value. What amount of depreciation expense should Spirit report in its financial statements for the year ending December 31, year 6? $2,800 $3,200 $43,200 $4,800

$2,800 This is a change in estimate and is handled currently and prospectively by allocating the remaining book value at the beginning of year 6 over the revised estimate of remaining years at that point. Through the beginning of year 6, the asset has been used five years. Therefore, seven years remain in the original book value. The book value at the beginning of year 6 is 7/12 × $48,000 or $28,000. The remaining useful life of seven years is extended to 10. Therefore, depreciation expense for year 6 is $28,000 × 1/10 or $2,800.

On June 1, 2005, Pitt Corp. sold merchandise with a list price of $5,000 to Burr on account. Pitt allowed trade discounts of 30% and 20%. Credit terms were 2/15, n/40 and the sale was made FOB shipping point. Pitt prepaid $200 of delivery costs for Burr as an accommodation. On June 12, 2005, Pitt received from Burr a remittance in full payment amounting to

$2,944 $5,000(1 - .30)(1 - .20)(.98) + $200 = $2,944.

On November 1, 2004, Davis Co. discounted with recourse at 10%, a one-year, noninterest-bearing, $20,500 note receivable maturing on January 31, 2005. What amount of contingent liability for this note must Davis disclose in its financial statements for the year ended December 31, 2004? $0 $20,000 $20,333 $20,500

$20,500 The firm is contingent for the maturity amount, which for a noninterest-bearing note is the face value. If the maker of the note fails to pay the bank or financial institution with whom Davis discounted the note, Davis would be called on to pay the entire maturity amount.

Talton Co. installed new assembly line production equipment at a cost of $185,000. Talton had to rearrange the assembly line and remove a wall to install the equipment. The rearrangement cost was $12,000 and the wall removal cost was $3,000. The rearrangement did not increase the life of the assembly line but it did make it more efficient. What amount of these costs should be capitalized by Talton? $185,000 $188,000 $197,000 $200,000

$200,000 This response includes all the costs to get the equipment ready for use. The rearrangement costs and the wall removal costs were needed to put the equipment into use. The rearrangement costs made the production more efficient.

Light Co. had the following bank reconciliation at March 31: Balance per bank statement, 3/31 $23,250 Add: Deposit in transit 5,150 28,400 Less: Outstanding checks 6,300 Balance per books, 3/31 $22,100 Additional information from Light's bank statement for the month of April is as follows: Deposits $29,200 Disbursements 24,800 All reconciling items at March 31 cleared through the bank in April. Outstanding checks at April 30 totaled $3,200. What is the amount of cash disbursements per books in April?

$21,700 disbursements per books is calculated as follows: April disbursements per bank statement $24,800 March checks cleared in April statement (6,300) Checks outstanding end of April 3,200Total cash disbursements in April $21,700

Carver Co., a retailer, uses the perpetual inventory method. Carver uses the moving average method to determine the value of its inventory. The following information relates to inventory transactions that took place during the month of March: 3/1 Beginning inventory 30,000 units at $10 3/5 Purchase 10,000 units at $12 3/10 Sales at $20 per unit20,000 units 3/20Purchase20,000 units at $13 What amount should Carver report as cost of goods sold on its income statement at the end of March? $200,000 $210,000 $240,000 $260,000

$210,000 Correct! The moving averages requires the calculation of the cost per unit based on the weighted average of the beginning inventory plus the purchase on March 5. At the date of the sale, the cost per unit is $10.50: total cost $420,000 ((30,000 × $10) + (10,000 × $12)) divided by the number of units 40,000 (30,000 + 10,000). Cost of goods sold is $210,000 (20,000 units sold × $10.50 price per unit).

Zahn Corp.'s comprehensive Balance Sheet at December 31, 2005 and 2004 reported accumulated depreciation balances of $800,000 and $600,000, respectively. Property with a cost of $50,000 and a carrying amount of $40,000 was the only property sold in 2005. Depreciation charged to operations in 2005 was: $190,000 $200,000 $210,000 $220,000

$210,000 The accumulated depreciation on the property sold was $10,000 ($50,000 cost less $40,000 carrying value). The sale of property requires that the accumulated depreciation on the property be removed from the accounts. Thus, the $10,000 amount is a decrease in accumulated depreciation. With an overall increase of $200,000 in accumulated depreciation during the period ($800,000-$600,000), depreciation must have been $210,000 ($200,000 + $10,000).

The following is Gold Corp.'s June 30 trial balance: Cash overdraft$ 10,000Accounts receivable, net$ 35,000Inventory58,000Prepaid expenses12,000Land held for resale100,000Property, plant, and equipment, net95,000Accounts payable and accrued expenses32,000Common stock25,000Additional paid-in capital150,000Retained earnings83,000__________________$300,000$300,000================ Additional information: Checks amounting to $30,000 were written to vendors and recorded on June 29 resulting in a cash overdraft of $10,000. The checks were mailed on July 9. Land held for resale was sold for cash on July 15. Gold issued its financial statements on July 31. In its June 30 balance sheet, what amount should Gold report as current assets?

$225,000 Current assets are those assets expected to be consumed or realized in cash within one year of the balance sheet date. There is no overdraft because the checks were not sent as of the balance sheet date. Thus, the balance sheet should disclose $20,000 in cash ($30,000 − $10,000). The land held for resale is a current asset because it is expected to be sold in the next year (and the corroboration of this expectation was known before the issuance of the financial statements). Cash$ 20,000 Net accounts receivable 35,000 Inventory 58,000 Prepaid expenses 12,000 Land held for resale 100,000 Total current assets $225,000

Star Corp. had the following accounts and balances in its general ledger as of December 31: Petty cash$ 500XYZ Bank checking account20,000Marketable equity security10,000Marketable debt security7,500ABC Bank depository account5,000 What amount should Star report as cash and cash equivalents in the balance sheet as of December 31?

$25,500

At the end of the year, Ian Co. determined its inventory to be $258,000 on a LIFO (last in, first out) basis. The current replacement cost of this inventory was $230,000. Ian estimates that it could sell the inventory for $275,000 at a disposal cost of $14,000. If Ian's normal profit margin for its inventory was $10,000, what would be its net carrying value? $244,000 $251,000 $258,000 $261,000

$251,000 The "ceiling" for LCM (lower of cost or market) valuation is $261,000 net realizable value ($275,000 selling price less $14,000 disposal cost). The "floor" is net realizable value less normal profit margin or $251,000 ($261,000 − $10,000). Replacement cost of $230,000 is below the floor so "market" value is the floor, or middle, of the three amounts ($251,000). This amount is less than cost of $258,000. Therefore, the lower of cost or market valuation is $251,000.

The following are held by Smite Co.: Cash in checking account$20,000Cash in bond sinking fund account30,000Postdated check from customer dated one month from balance sheet date250Petty cash200Commercial paper (matures in two months)7,000Certificate of deposit (matures in six months)5,000 What amount should be reported as cash and cash equivalents on Smite's balance sheet?

$27,200 The cash balance is $20,200: the sum of the checking account balance and the petty cash. Because it has a maturity of less than three months, the only cash equivalent is the $7,000 of commercial paper. The final sum of these two accounts is $27,200.

Spiro Corp. uses the sum-of-the-years' digits method to depreciate equipment purchased in January 20X3 for $20,000. The estimated salvage value of the equipment is $2,000, and the estimated useful life is four years. What should Spiro report as the asset's carrying amount as of December 31, 20X5? $1,800 $2,000 $3,800 $4,500

$3,800 The carrying amount (book value) of a depreciable asset is its original cost less accumulated depreciation. Under sum-of-the-years' digits method of calculating depreciation expense (and, therefore, accumulated depreciation), the net depreciable cost (original cost less estimated salvage value) is multiplied by a factor consisting of: Numerator = the number of years the current year is from the end of the life of the asset Denominator = the sum of numbers (digits) for each year in the life of the asset For Spiro, the net depreciable cost is $20,000-$2,000 = $18,000. Since the equipment has an estimated useful life of four years, the sum of the digits for each year would be 1 + 2 + 3 + 4 = 10, the denominator for calculating each year's depreciation. Depreciation for the four years would be: YearDepreciable costFactorAnnual depreciationAccumulated depreciationCarrying value20X3$18,000x 4/10=$7,200$ 7,200$20,000 -7,200=$12,80020X418,000x 3/10=5,40012,60020,000-12,600=7,40020X518,000x 2/10=3,60016,20020,000 -16,200=3,80020X618,000x 1/10=1,80018,00020,000 -18,000=2,000Total18,000x 10/10=18,00018,0002,000 Thus, at the end of 20X5 the carrying amount is $3,800, which also can be calculated as salvage value 2,000 + (1/10 × $18,000) = $2,000 + $1,800 = $3,800.

The following accounts were abstracted from Roxy Co.'s unadjusted trial balance on December 31, 2005: Debit Credit Accounts receivable $1,000,000 Allowance for uncollectible accounts 8,000 Net credit sales $3,000,000 Roxy estimates that 3% of the gross accounts receivable will become uncollectible. After adjustment at December 31, 2005, the allowance for uncollectible accounts should have a credit balance of $90,000 $82,000 $38,000 $30,000

$30,000 The correct answer, $30,000, is the ending balance in the allowance account and equals: .03($1,000,000) = $30,000. The firm is basing its estimate on receivables. Therefore, the $30,000 is the target ending allowance balance. The firm will recognize an amount of bad debt expense at the end of 2005 such that, after considering the beginning allowance balance and any write-offs during the year, the ending allowance balance will be $30,000. The bad debt expense amount recognized will be affected by the $8,000 beginning balance, but the ending allowance balance will not.

Ichor Co. reported equipment with an original cost of $379,000 and $344,000 and accumulated depreciation of $153,000 and $128,000, respectively, in its comparative financial statements for the years ended December 31, 20X5 and 20X4.During 20X5, Ichor purchased equipment costing $50,000 and sold equipment with a carrying value of $9,000. What amount should Ichor report as depreciation expense for 20X5? $19,000 $25,000 $31,000 $34,000

$31,000 Net equipment at end of 20X4: $344,000-$128,000 =$216,000 Equipment purchase 50,000 Book value of equipment sold (9,000) Depreciation in 20X5? Equals net equipment at end of 20X5: $379,000-$153,000 = $226,000 Solving for depreciation yields $31,000 depreciation for 20X5.

Alton Co. had a cash balance of $32,300 recorded in its general ledger at the end of the month, prior to receiving its bank statement. Reconciliation of the bank statement reveals the following information: Bank service charge: $15 Check deposited and returned for insufficient funds check: $120 Deposit recorded in the general ledger as $258 but should be $285 Checks outstanding: $1,800 After reconciling its bank statement, what amount should Alton report as its cash account balance?

$32,192 Correct! The reconciliation should be as follows: Book balance $32,300 Less bank fees (15) Less NSF check (120) Plus deposit transposition error (285 - 258) 27 Corrected book balance $32,192

On December 1, 20X5, East Co. purchased a tract of land as a factory site for $300,000. The old building on the property was razed and salvaged materials resulting from demolition were sold. Additional costs incurred and salvage proceeds realized during December 20X5 were as follows: Cost to raze old building$25,000Legal fees for purchase contract and to record ownership5,000Title guarantee insurance6,000Proceeds from sale of salvaged materials4,000 In East's December 31, 20X5 Balance Sheet, what amount should be reported as land? $311,000 $321,000 $332,000 $336,000

$332,000 The correct answer, $332,000, equals: $300,000 + $25,000-$4,000 + $5,000 + $6,000.The net cost to raze the old building ($21,000) is capitalized to land because it is a cost necessary to bring the land into its intended condition. The legal fees and title guarantee cost, likewise, must be incurred to avoid future legal problems, and thus contribute to the value of the land.

The following two inventory items were purchased as a group in a liquidation sale for $1,000. Replacement Carrying Value Item Cost On Seller's Books A$400 $390 B700 755 The firm purchasing the inventory records item A at what amount? $341 $390 $364 $500

$364 When items are purchased as a group, the total cost of the group is allocated to the individual items based on fair value. Replacement cost is the appropriate value to use in this case. The total replacement cost of the items is $1,100 ($400 + $700). Therefore, Item A is allocated 4/11 of the purchase cost, or $364 = ($400/$1,100)$1,000.

South Co. purchased a machine that was installed and placed in service on January 1, 20X4 at a cost of $240,000. Salvage value was estimated at $40,000. The machine is being depreciated over 10 years by the double declining balance method. For the year ended December 31, 20X5, what amount should South report as depreciation expense? $48,000 $38,400 $32,000 $21,600

$38,400 Depreciation in 2004 = $240,000(2/10) = $48,000Depreciation in 20X5 = ($240,000-$48,000)(2/10) = $38,400 The DDB method's rate is always twice the straight-line rate, or 2/useful life. The method does not subtract salvage value when computing depreciation, but it also does not reduce book value below salvage value. The depreciation in any year is the rate times the beginning net book value of the asset.

Moss Co. has determined its December 31, 20X4 inventory to be $400,000 on a FIFO basis. Information pertaining to that inventory follows: Estimated selling price $408,000 Estimated cost of disposal 20,000 Normal profit margin 60,000 Current replacement cost 360,000 Moss records losses that result from applying lower of cost or net realizable value. On December 31, 20X4, what should be the net carrying value of Moss' inventory? $400,000 $388,000 $360,000 $328,000

$388,000 Lower of cost or net realizable value applies to inventories that are carried at FIFO or Average cost. Net realizable value is selling price less cost of disposal. In this case it is $408,000 − $20,000 = $388,000.

A firm uses the dollar value LIFO retail method and has $2,000 in beginning inventory at retail at the beginning of the current year. The base year equivalent of this amount is $1,600. The base year index is 1.00. The beginning inventory reported in the Balance Sheet is $800. During the current year, the firm purchased $12,000 of inventory at cost and marked that up to $40,000. Sales for the year were $28,000. The relevant ending price index is 1.60. What amount does this firm report as inventory in its Balance Sheet at the end of the current year? $4,286 $13,440 $4,232 $4,200

$4,232 This is a two-step process. First, DV LIFO is applied to retail dollars to determine the layer added in current-year retail dollars. Then, the FIFO cost-to-retail ratio (C/R) is applied to convert that layer to cost. Finally, this layer is added to beginning inventory at cost to yield ending inventory at cost. The calculation is: EI retail, current index = $2,000 + $40,000-$28,000 = $14,000 EI retail, base = $14,000/1.6 = $8,750 Increase in EI retail, base = $8,750-$1,600 = $7,150 Increase in EI retail, current = $7,150(1.6) = $11,440 C/R (use FIFO, not LCM) = $12,000/$40,000 = .30 Increase in EI, cost = .30($11,440) = $3,432 EI, cost = $800 + $3,432= $4,232

On March 31, 2005, Vale Co. had an unadjusted credit balance of $1,000 in its allowance for uncollectible accounts. An analysis of Vale's trade accounts receivable on that date revealed the following: Age Amount Estimated uncollectible 0 − 30 days $60,000 5% 31 − 60 days 4,000 10% Over 60 days 2,000 $1,400 What amount should Vale report as allowance for uncollectible accounts in its March 31, 2005, balance sheet?

$4,800 $60,000(5%) + $4,000(10%) + $1,400 = $4,800.

The following information applied to Fenn, Inc. for 2005: Merchandise purchased for resale$400,000Freight-in10,000Freight-out5,000Purchase returns2,000 Fenn's 2005 inventoriable cost was $400,000 $403,000 $408,000 $413,000

$408,000 Merchandise purchased for resale $400,000 Freight-in 10,000 Purchase returns (2,000) Total inventoriable cost $408,000 Freight-out is a delivery expense. It is not inventoried because the goods have reached salable condition before incurring this cost. Only costs that contribute to preparing inventory for sale are inventoried.

In 2005, Cobb adopted the dollar-value LIFO inventory method. At that time, Cobb's ending inventory had a base-year cost and an end-of-year cost of $300,000. In 2006, the ending inventory had a $400,000 base-year cost and a $440,000 end-of-year cost. What dollar-value LIFO inventory cost would be reported in Cobb's December 31, 2006, balance sheet? $440,000 $430,000 $410,000 $400,000

$410,000 The price level index for 2006 is 1.1 ($440,000/$400,000). Ending 2006 DV LIFO inventory equals the beginning inventory DV LIFO plus the increase in inventory at base-year dollars converted to 2006 prices: Ending DV LIFO = Beginning DV LIFO + (increase at base-year dollars)(1.1)= $300,000 + ($400,000 − $300,000)(1.1) = $410,000.

The following items were included in Opal Co.'s inventory account on December 31, 2004: Merchandise out on consignment, at sales price, including 40% markup on selling price$40,000Goods purchased, in transit, shipped FOB shipping point36,000Goods held on consignment by Opal27,000 By what amount should Opal's inventory account at December 31, 2004 be reduced? $103,000 $67,000 $51,000 $43,000

$43,000 The merchandise out on consignment is included in inventory at selling price. But inventory must be measured at cost. $40,000 = cost + .40($40,000). Thus, cost = $24,000. Therefore, inventory should be reduced by the $16,000 of markup on the merchandise out on consignment. The goods held on consignment should be removed from the inventory because these goods do not belong to Opal. Hence, the total reduction from inventory is $43,000 ($16,000 + $27,000). The goods in transit are properly included in inventory because they were shipped FOB shipping point, which means the goods belong to Opal when the goods reach the common carrier at the shipping point.

On December 1, 20X5, Boyd Co. purchased a $400,000 tract of land for a factory site. Boyd razed an old building on the property and sold the materials it salvaged from the demolition. Boyd incurred additional costs and realized salvage proceeds during December 20X5 as follows: Demolition of old building$50,000Legal fees for purchase contract and recording ownership10,000Title guarantee insurance12,000Proceeds from sale of salvaged materials8,000 In its December 31, 20X5, Balance Sheet, Boyd should report a balance in the land account of: $464,000 $460,000 $442,000 $422,000

$464,000 Land purchase price $400,000 Plus demolition of old building 50,000 Less salvage proceeds (8,000) Plus title insurance 12,000 Plus legal fees 10,000 Equals recorded land cost $464,000 The net demolition cost is included in land because it is a cost required to prepare the land for its eventual use.

Poe, Inc. had the following bank reconciliation at March 31 Balance per bank statement, March 31 $46,500 Add deposit in transit 10,300 56,800 Less outstanding checks 12,600 Balance per books, March 31 $44,200 Data per bank for the month of April follow: Deposits $58,400 Disbursements 49,700 All reconciling items at March 31 cleared the bank in April. Outstanding checks at April 30 totaled $7,000. There were no deposits in transit at April 30. What is the cash balance per books at April 30?

$48,200 Balance per books, 3/31 $44,200 Deposits per bank, April $58,400 Less deposit in transit, 3/31 (10,300) Equals deposits made by firm in April 48,100 Checks clearing bank in April $49,700 Less outstanding checks, 3/31 (12,600) Plus outstanding checks, 4/30 7,000 Equals checks written by firm in April (44,100) Balance per books, 4/30 $48,200

A firm's inventory was destroyed by fire on August 14 of the current year. Fortunately, the firm had insurance to cover the loss. However, most of the inventory records were also destroyed in the fire. The average gross margin percentage is 40%, beginning inventory was $200,000, and $1,000,000 of purchases had been made through August 13. The firm had recorded sales of $1,200,000 through that date. Estimate the cost of the inventory lost in the fire. $0 $480,000 $720,000 $280,000

$480,000 The gross margin method estimates the cost of inventory at the time of the fire as follows: Beginning inventory $200,000 + $1,000,000 Purchases = Ending inventory + Cost of goods sold. The estimate of cost of goods sold is found by multiplying the cost to sales ratio and sales. The gross margin percentage plus the cost to sales ratio is 1; therefore, cost/sales is .60 (= 1 - .40). Estimated cost of goods sold is $720,000 (= .60($1,200,000)). The equation is Beginning inventory $200,000 + Purchases $1,000,000 = Ending inventory + Cost of goods sold $720,000. Solving for ending inventory yields $480,000.

Ace Co. sold King Co. a $20,000, 8%, 5-year note that required five equal annual year-end payments. This note was discounted to yield a 9% rate to King. The present value factors of an ordinary annuity of $1 for five periods are as follows: 8% 3.992 9% 3.890 What should be the total interest revenue earned by King on this note? $9,000 $8,000 $5,560 $5,050

$5,560 Total interest over the life of the note equals the total amount paid by Ace over the life of the note less the proceeds to Ace. The proceeds equal the present value of the payments at the 9% yield rate. The annual payment is found using the 8% rate because that rate is contractually set and determines the annual payment. The annual payment P is found as: $20,000 = P(3.992). P = $5,010 Total interest revenue = total payments by Ace - proceeds to Ace= 5($5,010) − $5,010(3.89) = $5,560.

Leaf Co. purchased from Oak Co. a $20,000, 8%, 5-year note that required five equal annual year-end payments of $5,009. The note was discounted to yield a 9% rate to Leaf. At the date of purchase, Leaf recorded the note at its present value of $19,485. What should be the total interest revenue earned by Leaf over the life of this note? $5,045 $5,560 $8,000 $9,000

$5,560 Total interest revenue is the amount received over the term of the note less the present value of the note: 5($5,009) − $19,485 = $5,560. Leaf paid $19,485 for the note, and will receive 5($5,009) over the note term. The difference is interest revenue.

On December 28, 2005, Kerr Manufacturing Co. purchased goods costing $50,000. The terms were FOB destination. Some of the costs incurred in connection with the sale and delivery of the goods were as follows: Packaging for shipment$1,000 Shipping 1,500 Special handling charges 2,000 These goods were received on December 31, 2005. In Kerr's December 31, 2005 balance sheet, what amount of cost for these goods should be included in inventory? $54,500 $53,500 $52,000 $50,000

$50,000 Kerr will pay only $50,000 for the goods. None of the other costs listed are incurred by Kerr. Rather, the seller will incur those costs. Even the shipping costs are borne by the seller because the terms are FOB destination. This means that title does not transfer to the buyer (Kerr) until the goods reach the destination. The seller owned the goods in transit and therefore incurred the transportation cost. Kerr's recorded cost is $50,000.

Ott Co. purchased a machine at an original cost of $90,000 on January 2, Year 1. The estimated useful life of the machine is 10 years, and the machine has no salvage value. Ott uses the straight-line method to calculate depreciation. On July 1, Year 10, Ott sold the machine for $5,000. What is the amount of gain or loss on the disposal of the machine? $500 loss $500 gain $4,500 loss $4,500 gain

$500 gain Correct! The gain or loss on the disposal of an asset is the difference between the net book value (NBV) and the selling price. The annual depreciation of the machine is $9,000 ($90,000/10 years). The asset was held for 9.5 years, so accumulated depreciation is $85,500. NBV is $4,500 ($90,000 less accumulated depreciation of $85,500). The sale is a gain of $500 because the selling price is greater than the NBV ($5,000 − $4,500).

Delar Co. completed its year-end physical count of inventory. The inventory was valued at first-in, first-out (FIFO) costs and totaled $500,000. Delar subsequently noted the following two items: 1,000 units of inventory with a FIFO cost of $10 each were shipped and billed to a customer, f.o.b. destination. These items were included in the physical count. 6,000 units at a FIFO cost of $5 each were held on consignment for one of its suppliers, but were excluded from the physical count. What amount should Delar report as inventory at year end? $530,000 $520,000 $500,000 $490,000

$500,000 Correct! Goods on consignment are excluded from ending inventory and goods shipped f.o.b. destination are included. There is no adjustment need to the $500,000 inventory value.

Roth, Inc. received from a customer a one-year, $500,000 note bearing annual interest of 8%. After holding the note for six months, Roth discounted the note at Regional Bank at an effective interest rate of 10%. What amount of cash did Roth receive from the bank? $540,000 $523,810 $513,000 $495,238

$513,000 Maturity value of the note: $500,000(1.08)$540,000Less discount to the bank: $540,000(.10)(6/12)(27,000)Equals proceeds to Roth$513,000 The bank charges its discount on the maturity amount, for the period it holds the note. In effect, it is charging interest on interest yet to accrue (for the last six months). This procedure is followed because the maturity value is the amount at risk.

Hilltop Co.'s monthly bank statement shows a balance of $54,200. Reconciliation of the statement with company books reveals the following information: Bank service charge $10 Insufficient funds check 650 Checks outstanding 1,500 Deposits in transit 350 Check deposited by Hilltop and cleared by the bank for $125 but improperly recorded by Hilltop as $152 What is the net cash balance after the reconciliation?

$53,050 The reconciling items that need to be adjusted to the bank balance are: checks outstanding (−1,500) and deposit in transit (+350). The net cash after the reconciliation is: Bank balance $54,200 − 1,500 + 350 = $53,050. The bank service charge and insufficient funds are already reflected in the bank balance. The error is on Hilltop's books, not on the bank statement, and therefore it does not need to be included in the reconciliation.

A flash flood swept through Hat, Inc.'s warehouse on May 1. After the flood, Hat's accounting records showed the following: Inventory, January 1$ 35,000Purchases, January 1 through May 1200,000Sales, January 1 through May 1250,000Inventory not damaged by flood30,000Gross profit percentage on sales40% What amount of inventory was lost in the flood? $55,000 $85,000 $120,000 $150,000

$55,000 The gross margin method of estimating inventory is used to solve this problem. The cost of inventory lost cannot be identified by count but it can be estimated. First, an estimate of cost of goods sold is subtracted from the cost of goods available on the date of the flood yielding the total amount of inventory that would have been present on May 1. Second, the amount of inventory not lost is subtracted from the May 1 estimated total inventory. The result is an estimate of the amount lost. With gross profit being 40% of sales, cost of goods sold must be 60% of sales, on average. Therefore, the estimate of cost of goods sold is $150,000 (.60 × $250,000). Beginning inventory ($35,000) + Purchases ($200,000) = Goods available = $235,000. Subtracting $150,000 of cost of goods sold yields $85,000 of inventory on May 1 ($235,000 − $150,000). With $30,000 of inventory still accounted for, the amount of lost inventory at cost is $55,000 ($85,000 − $30,000).

For the year ending December 31, 2005, Beal Co. estimated its allowance for uncollectible accounts using the year-end aging of accounts receivable. The following data are available: Allowance for uncollectible accounts, 1/1/05 $42,000 Provision for uncollectible accounts during 2005 (2% on credit sales of $2,000,000) 40,000 Uncollectible accounts written off, 11/30/05 46,000 Estimated uncollectible accounts per aging, 12/31/05 52,000 After year-end adjustment, the uncollectible accounts expense for 2005 should be

$56,000 The balance in the allowance for uncollectible accounts before the 2005 adjustment is: $42,000 beginning balance − $46,000 write-offs = -$4,000 (debit balance). The desired ending balance under aging is $52,000.

A firm has spent the last two years constructing a building to be used as the firm's headquarters. At the end of the first year of construction, the balance of building under construction was $400,000, which includes capitalized interest. During year two, the firm paid $240,000 to the contractor on March 1, and $600,000 on October 1. The building was not finished by the end of the second year. The firm had one loan outstanding all year, an 8%, $3,000,000 construction loan. Compute capitalized interest for year two. $28,000 $240,000 $60,000 $65,600

$60,000 Average accumulated expenditures for the second year = $400,000(12/12) + $240,000(10/12) + $600,000(3/12) = $750,000. Interest capitalized = .08($750,000) = $60,000. Note that the interest capitalized in year one is compounded in year two because year one capitalized interest is included in average accumulated expenditures for the second year.

Each of Potter Pie Co.'s 21 new franchisees contracted to pay an initial franchise fee of $30,000.By December 31, year 5, each franchisee had paid a nonrefundable $10,000 fee and signed a note to pay $10,000 principal plus the market rate of interest on December 31, year 6 and December 31, year 7.Experience indicates that one franchise will default on the additional payments. Services for the initial fee will be performed in year 6. What amount of net unearned franchise fees would Potter report on December 31, year 5? $400,000 $600,000 $610,000 $630,000

$610,000 The $610,000 net unearned fee revenue = (21)($30,000) − $20,000. This amount includes the notes received, but does not include the one expected uncollectible note. The notes receivable balance, recorded along with the unearned revenue, will not reflect the note expected to be uncollectible. Bad debt expense is not recorded for this note because there has been no revenue recognized against which to match the expense.

On December 30, 2005, Chang Co. sold a machine to Door Co. in exchange for a noninterest-bearing note requiring ten annual payments of $10,000. Door made the first payment on December 30, 2005. The market interest rate for similar notes at the date of issuance was 8%. Information on present value factors is as follows: Period Present value of $1 at 8% Present value of ordinary annuity of $1 at 8% 9 0.50 6.25 10 0.46 6.71 In its December 31, 2005, balance sheet, what amount should Chang report as note receivable? $45,000 $46,000 $62,500 $67,100

$62,500 The note receivable should be reported at the present value of the nine remaining payments. The first payment was made at the date of the sale. The remaining nine payments comprise an ordinary annuity as of December 31, 2005 because the next payment is due one year from that date. Therefore, the present value and reported note value on that date is 6.25($10,000) = $62,500.

Brock Co. adopted the dollar-value LIFO inventory method as of January 1, 2003. A single inventory pool and an internally computed price index are used to compute Brock's LIFO inventory layers. Information about Brock's dollar-value inventory follows: Inventory Date at base-year cost at current-year cost at dollar-value LIFO 1/1/03 $40,000 $40,000 $40,000 2003 layer 5,000 14,000 6,000 ________________________________________ 12/31/03 45,000 54,000 46,000 2004 layer 15,000 26,000?________________________________________ 12/31/04 $60,000 $80,000?============================= What was Brock's dollar-value LIFO inventory on December 31, 2004? $80,000 $74,000 $66,000 $60,000

$66,000 The ending inventory is used to construct the internal price index. At the end of 2004, the ratio of current cost to base-year cost for ending inventory is $80,000/$60,000 = 1 1/3 or 4/3. This ratio is applied to the 2004 layer at base-year cost, yielding $20,000 ($15,000 × 4/3). This amount is the increase to DV LIFO. Therefore, ending 2004 DV LIFO is $66,000 ($46,000 + $20,000).

On April 1, Aloe, Inc. factored $80,000 of its accounts receivable without recourse. The factor retained 10% of the accounts receivable as an allowance for sales returns and charged a 5% commission on the gross amount of the factored receivables. What amount of cash did Aloe receive from the factored receivables? $68,000 $68,400 $72,000 $76,000

$68,000 The net cash received when the receivables were factored was $80,000 × .85 (100% - 10% - 5%) = $68,000.

At the beginning of the year, Cann Co. started construction on a new $2 million addition to its plant. Total construction expenditures made during the year were $200,000 on January 2, $600,000 on May 1, and $300,000 on December 1. On January 2, the company borrowed $500,000 for the construction at 12%. The only other outstanding debt the company had was a 10% interest rate, long-term mortgage of $800,000, which had been outstanding the entire year. What amount of interest should Cann capitalize as part of the cost of the plant addition? $140,000 $132,000 $72,500 $60,000

$72,500 First calculate the Average Accumulated Expenditures (AAE). This gives you the amount of borrowing from which to calculate avoidable interest ($625,000). Next calculate avoidable interest ($72,500) and actual interest (($500,000 × 12%) + ($800,000 × 10%) = $140,000). The amount that can be capitalized is the lesser of the avoidable interest or actual interest. The amount that can be capitalized is $72,500. AAE 200,000 12/12 200,000 600,000 8/12 400,000 300,000 1/12 25,000 625,000 Avoidable interest 500,000 12% 60,000 125,000 10% 12,500 72,500

Papa Company acquired land with an office building on it from its subsidiary, Sonny Company, for $110,000. Prior to the sale, Sonny's carrying value of the land was $60,000 and its net carrying value of the building was $50,000. At the time of the transaction, Papa appropriately determined that the land had a fair value of $75,000 and the building had a fair value of $35,000. At what amount should Papa record the land and building on its books at the date of the transaction? Land Building $75,000 $35,000 $55,000 $55,000 $60,000 $50,000 $50,000 $60,000

$75,000 $35,000 Papa should record the land and building on its books at the appropriately determined fair value at the date of the transaction. The prior carrying values on Sonny's books are not relevant to the amounts at which Papa should record the assets on its books, but are relevant to the amounts that should be reported in the consolidated financial statements.

Frame Co. has an 8% note receivable, in the original amount of $150,000, dated June 30, 2003. Payments of $50,000 in principal plus accrued interest are due annually on July 1, 2004, 2005, and 2006. In its June 30, 2005, balance sheet, what amount should Frame report as a current asset for interest on the note receivable? $0 $4,000 $8,000 $12,000

$8,000 As of June 30, 2005, only one payment has been received (July 1, 2004). Thus, $100,000 of principal balance has been outstanding for an entire year as of the balance sheet date. Interest receivable on June 30, 2005 is thus $8,000 (.08 × $100,000).

A firm began the construction of its new manufacturing facility in January of 20x2. The following expenditures were made on construction in that year: Jan. 1 $40,000 Mar. 1 120,000 Oct. 3 196,000 Debt outstanding the entire year: 6%, $60,000 construction loan4%, $90,000 note payable not related to construction6%, $90,000 note payable not related to construction Compute interest to be capitalized using the weighted average method. $6,720 $12,600 $8,400 $8,190

$8,190 This Answer is Correct Average accumulated expenditures is $156,000 = $40,000 + $120,000(10/12) + $96,000(2/12). This method uses the average interest rate on all interest bearing debt, weighted by principal. That rate is the quotient of the interest on all the debt divided by the principal on all the debt. The rate = ($3,600 + $3,600 + $5,400)/$240,000 = .0525. Interest capitalized = (.0525)$156,000 = $8,190.

In January, Stitch, Inc. adopted the dollar-value LIFO method of inventory valuation. At adoption, inventory was valued at $50,000. During the year, inventory increased $30,000 using base-year prices, and prices increased 10%. The designated market value of Stitch's inventory exceeded its cost at year-end. What amount of inventory should Stitch report in its year-end balance sheet? $80,000 $83,000 $85,000 $88,000

$83,000 Beginning inventory of $50,000 is at base-year dollars and the current year increase of $30,000 is also at base-year dollars. The current year layer must be converted to current year costs ($30,000 × 1.10) = $33,000. Ending dollar value LIFO is the beginning dollar value LIFO (in this case it was adopted in January so the beginning inventory must be $50,000) plus the current year layer of $33,000 or $83,000. Note that the sentence "The designated market value of Stitch's inventory exceeded its cost at year end" is a distracter. It is simply stating that there is not an issue with the lower of cost or market since cost is lower.

Beck Co.'s inventory of trees is as follows: Beginning Inventory10 trees @ $ 50 March 4 purchased 6 trees @ 55 March 12 sold 8 trees @ 100 March 20 purchased 9 trees @ 60 March 27 sold 7 trees @ 105 March 30 purchased 4 trees @ 65 What was Beck's cost of goods sold using the last in, first out (LIFO) perpetual method? $910 $850 $808 $775

$850 Correct! The perpetual method recognizes each purchase and sale at the time it occurs. The total value of cost of sales using perpetual LIFO is ((6 × $55) + (2 × $50) + (7 × $60)) = $850, as shown in the table below. Trees remaining Cost of trees sold Begin Inv10 @ $ 5010 @ $50 March 4 purchased6 @ 55 10 @ $50 6 @ 55 March 12 sold 8 @ 100 8 @ 50 6 @ $55 = $330 2 @ $50 = 100 March 20 purchased 9 @ 60 8 @ 50 9 @ 60 March 27 sold 7 @ 105 8 @ 50 2 @ 60 7 @ $60 = 420 March 30 purchased 4 @ 65 8 @ 50 2 @ 60 4 @ 65 Total cost = $850

The following information pertains to Tara Co.'s accounts receivable on December 31, Year 4 Days outstandingAmountEstimated % uncollectible 0-60 $120,000 1% 61-120 90,000 2% Over 120 100,000 6% $310,000======== During Year 4, Tara wrote off $7,000 in receivables and recovered $4,000 that had been written off in prior years. Tara's December 31, Year 3, allowance for uncollectible accounts was $22,000. Under the aging method, what amount of allowance for uncollectible accounts should Tara report on December 31, Year 4?

$9,000 The data on write-offs and recoveries is not relevant. The aging method computes a required ending allowance balance based on the aging schedule. That required ending balance is the sum of the products of the receivables in each age category and the uncollectible percentage: $120,000(.01) + $90,000(.02) + $100,000(.06) = $9,000. The write-offs and recoveries do affect the preadjustment allowance balance and therefore the amount of uncollectible accounts expense to recognize. In this case, the preadjustment balance is $19,000 ($22,000 − $7,000 + $4,000), which means no uncollectible accounts expense would be recognized in 2004 because the preadjustment balance is more than sufficient (exceeds the $9,000 required balance).

Ashe Co. recorded the following data pertaining to raw material X during January 2005: Units Date Received Cost Issued On Hand 1/1/05 Inventory $8.00 3,200 1/11/05 Issue 1,600 1,600 1/22/05 Purchase 4,800 $9.60 6,400 The moving-average unit cost of X inventory on January 31, 2005 is $8.80 $8.96 $9.20 $9.60

$9.20 Units beginning inventory remaining at year-end (3,200 - 1,600)$8 =$12,800Plus 1/22 purchase: 4,800($9.60) =46,080Ending inventory$58,880Ending unit cost: $58,880/6,400 =$9.20 The moving average method costs issues at the unit cost of goods on hand at that point. Thus, the issue was costed at $8.00 per unit. The cost per unit changes with each purchase.

Fountain Co. is constructing an office building for its own use. Fountain started the two-year construction project on January 1, year 1, at which point the interest capitalization period began. Fountain made the following payments in year 1 related to the construction of the building: April 1—Payment to architect for building plans:$30,000July 1—Progress payment to contractor:60,000October 1—Progress payment to contractor:150,000 For the purpose of capitalizing interest, what is Fountain's weighted average accumulated expenditures for the year ended December 31, year 1? $80,000 $90,000 $120,000 $240,000

$90,000 Correct! Interest is capitalized on expenditures only when (1) qualifying expenditures have been made, (2) construction is proceeding, and (3) interest cost is incurred. Construction started on January 1, year 1, thus the year 1 construction period is for 12 months. The calculation of the average accumulated expenditures is: (30,000 × 9/12) + (60,000 × 6/12) + (150,000 × 3/12) = 90,000.

The following information was taken from Cody Co.'s accounting records for the year ended December 31, 2005: Decrease in raw materials inventory$ 15,000Increase in finished goods inventory35,000Raw materials purchased430,000Direct labor payroll200,000Factory overhead300,000Freight-out45,000 There was no work-in-process inventory at the beginning or end of the year. Cody's 2005 cost of goods sold is $895,000 $910,000 $950,000 $955,000

$910,000 The correct answer is $910,000: Raw materials purchased $430,000 Plus decrease in raw materials 15,000* Direct labor 200,000 Factory overhead 300,000 Less finished goods increase (35,000) **Cost of goods sold $910,000 *The decrease in raw materials is added to the amount purchased resulting in the cost of materials incorporated into production. In other words, $15,000 of materials purchased in 2005 were placed into production in 2005. The total cost of materials brought into production in 2005 equals $445,000. ** The increase in finished goods represents costs incurred in the current period to finish inventory that was not sold in the current period. Therefore, these costs must be removed in determining cost of goods sold. Freight-out is not a manufacturing cost but rather is a distribution cost. Therefore, freight-out is not inventoried. There is no change in work-in-process inventory to affect the calculation.

On the December 31, year 5 balance sheet of Mann Co., the current receivables consisted of the following: Trade accounts receivable$ 93,000 Allowance for uncollectible accounts(2,000) Claim against shipper for goods lost in transit (Nov. year 5) 3,000 Selling price of unsold goods sent by Mann on consignment at 130% of cost (not included in Mann's ending inventory) 26,000 Security deposit on lease of warehouse used for storing some inventories 30,000 Total$150,000======== On December 31, year 5, the correct total of Mann's current net receivables was

$94,000 Only the first three items are included in net receivables: Trade accounts receivable $93,000 Allowance for uncollectible accounts (2,000) Claim against shipper for goods lost in transit (Nov. year 5) 3,000 Net receivables $94,000

A plant asset under construction by a firm for its own use was completed at the end of the current year. The following costs were incurred: Materials$60,000 Labor30,000 Incremental overhead10,000 Capitalized interest20,000 The asset has a service life of 10 years, estimated residual value of $10,000, and will be depreciated under the double declining balance method. At completion, the asset was worth $105,000 at fair value. What amount of depreciation will be recognized on the asset in total over its service life? $105,000 $120,000 $95,000 $90,000

$95,000 The sum of the four listed costs is $120,000, which exceeds fair value of $105,000. Therefore, the asset is capitalized at $105,000, the lesser of the two amounts. Subtracting the $10,000 residual value yields $95,000 depreciable cost-the total depreciation over the life of the asset.

On January 2, 2003, Emme Co. sold equipment with a carrying amount of $480,000 in exchange for a $600,000 noninterest-bearing note due January 2, 2006. There was no established exchange price for the equipment. The prevailing rate of interest for a note of this type on January 2, 2003, was 10%. The present value of $1 at 10% for three periods is 0.75. In Emme's 2003 income statement, what amount should be reported as gain (loss) on sale of machinery? ($30,000) loss. $30,000 gain. $120,000 gain. $270,000 gain.

($30,000) loss. The proceeds on sale are measured as the present value of the note because there is no established market value for the equipment. The loss on sale is computed as: Carrying amount $480,000 Less proceeds on sale: $600,000 (.75) =450,000 Equals loss on sale$ 30,000 The proceeds do not reflect the entire $600,000, because the difference between the note's face value of $600,000 and its present value is interest to be recognized over the term of the note.

Bach Co. adopted the dollar-value LIFO inventory method as of January 1, 2006. A single inventory pool and an internally computed price index are used to compute Bach's LIFO inventory layers. Information about Bach's dollar-value inventory follows: Inventory Date at base-year cost at current-year cost 1/1/06 $90,000 $90,000 2006 layer 20,000 30,000 2007 layer 40,000 80,000 What was the price index used to compute Bach's 2007 dollar-value LIFO inventory layer? 1.09 1.25 1.33 2.00

1.33 The question provides the ending inventory for 2007 at current cost by layer. The sum of the current cost column ($200,000) is the current cost of the entire inventory at the end of 2007. The sum of the base-year cost for the three years is $150,000. Hence, under this assumption, the ratio of current cost for the total inventory at the end of 2007 to the base-year cost is 1.33 ($200,000/$150,000). This index is then multiplied by the 2007 layer in base-year dollars to derive the increment to DV LIFO ending inventory.

A bank reconciliation with the headings "Balance per Books" and "Balance per Bank" lists three adjustments under the former and four adjustments under the latter. The company makes separate adjusting entries for each item in the reconciliation that requires an adjustment. How many adjusting entries are recorded?

3 Only amounts adjusting the balance per books require an adjusting entry because only those amounts explain why the firm's recorded cash balance is not the same as the true cash balance. Common adjustments of this type include bank service charges, notes collected, and interest. The firm cannot alter the bank balance.

When marking up a specific line of household items for resale, a retailer computes its markup as 40% of cost. For purposes of estimating ending inventory using the gross margin method, what percentage is applied to sales when estimating cost of goods sold? 40 71 60 29

71 The gross margin method applies the cost to sales ratio to sales in order to derive an estimate of cost of goods sold. Subtracting the resulting estimate of cost of goods sold from the cost of goods available for sale yields an estimate of ending inventory without counting the items. This firm determines the selling price to be 140% of cost because the markup is 40% of cost. Cost plus markup yields selling price. Therefore, the cost to sales ratio is 1.00/1.40 or .71.

Which of the following statements regarding inventory accounting systems is true? A disadvantage of the perpetual inventory system is that the inventory dollar amounts used for interim reporting purposes are estimated amounts. A disadvantage of the periodic inventory system is that the cost of goods sold amount used for financial reporting purposes includes both the cost of inventory sold and inventory shortages. An advantage of the perpetual inventory system is that the record keeping required to maintain the system is relatively simple. An advantage of the periodic inventory system is that it provides a continuous record of the inventory balance.

A disadvantage of the periodic inventory system is that the cost of goods sold amount used for financial reporting purposes includes both the cost of inventory sold and inventory shortages. A periodic system does not record the cost of each item as it is sold; nor does it maintain a continuously current record of the inventory balance. Rather, cost of goods sold is the amount derived from the equation: Beginning inventory + Purchases = Ending inventory + Cost of goods sold. A count of ending inventory establishes the inventory remaining at the end of the period, but there is no recording of cost of goods sold during the period. Cost of goods sold is the amount that completes the equation. Thus, cost of goods sold is really the cost of inventory no longer with the firm at year-end - an amount that includes shrinkage. Inventory shrinkage refers to breakage, waste, and theft. Shrinkage cannot be identified directly with a periodic inventory system.

In October of year one, a firm committed to a purchase of inventory at a total cost of $26,000. The contract is irrevocable and specifies a delivery date in March of year two. At the end of year one, the market value of the inventory under contract is worth $23,000 at current cost. Choose the correct reporting for the year one financial statements: A liability of $3,000 is reported in the Balance Sheet. An extraordinary loss of $3,000 is reported in the Income Statement. The potential loss on contract is reported in the footnotes, but there is no recognition in the financial statements. No reporting is required.

A liability of $3,000 is reported in the Balance Sheet. The firm has committed to a purchase for a total cost of $26,000, but at year-end, the value of the item to be received is $3,000 less. The firm cannot postpone the loss and liability recognition because the reduction in the firm's earnings and net assets has already occurred. The economic events causing the loss have occurred as of the Balance Sheet date.

When a note receivable is determined to be impaired, The note is written-off. No recognition of the impairment is required until a formal troubled-debt restructuring takes place. The note is written down to the nominal sum of future cash flows expected to be collected, including interest. A loss or expense is recognized as equal to the difference between the note carrying value and the present value of the cash flows expected to be received.

A loss or expense is recognized as equal to the difference between the note carrying value and the present value of the cash flows expected to be received. A note is considered to be impaired if the present value of remaining cash flows is less than book value, using the rate in the note. This is caused by an expected delay in timing of cash flows or reduction in amount of cash flows compared with the original agreement. The creditor makes the determination that the note is impaired and writes the note down to present value. A loss is recorded for the decline in carrying value to present value.

When an inventory overstatement in year one counterbalances in year two, this means: There are no reporting errors, even if the overstatement is never discovered. A prior period adjustment is recorded if the error is discovered in year three. The year one Balance Sheet does not need to be restated if the error is discovered in year three. A prior period adjustment is recorded if the error is discovered in year two.

A prior period adjustment is recorded if the error is discovered in year two. Counterbalancing simply means that the effect of the inventory error in the second year is opposite that of the first year. Discovery in year two provides an opportunity for the firm to correct year two beginning retained earnings, which is overstated by the error in year one. The overstatement of inventory in year one caused cost of goods sold to be understated and income overstated in year one. The prior period adjustment, dated as of the beginning of year two, is a debit to retained earnings for the after-tax effect of the income overstatement in year one. Inventory is credited for the amount of the overstatement. This allows year two to begin with corrected balances.

Gibbs Co. uses the allowance method for recognizing uncollectible accounts. Ignoring deferred taxes, the entry to record the write-off of a specific uncollectible account

Affects neither net income nor working capital.

Which of the following is a required footnote disclosure on property, plant, and equipment? Range of useful lives of plant assets. Depreciation methods of plant assets. Accumulated depreciation related to plant assets. All of the above.

All of the above. All items listed are required disclosures: useful life, depreciation methods, and the accumulated depreciation of plant asset. Read through select disclosures of the financial statements of real companies-this will help reinforce the disclosure requirements and jog your memory because you will remember reading about the disclosure.

In which of the following situations is the units of production method of depreciation most appropriate? An asset's service potential declines with use. An asset's service potential declines with the passage of time. An asset is subject to rapid obsolescence. An asset incurs increasing repairs and maintenance with use.

An asset's service potential declines with use. This method is most appropriate when the service potential of an asset can be estimated reliably in terms of a physical variable, such as miles to be driven, or number of units of output that can be produced by the asset. Over time, as more units are produced, the service potential of the asset declines because the total number of units that can be produced is finite. Over time, the number of units that can be produced by the asset in the future declines. The primary causative agent for depreciation under the units of production method is, thus, the actual use of the asset in production.

Units Unit cost Total cost Units on hand Balance on 1/1 2,000 $1 $2,000 2,000 Purchased on 1/8 1,200 33,600 3,200 Sold on 1/2 3 1,800 1,400 Purchased on 1/28 800 5 4,000 2,200 Nest uses the LIFO method to cost inventory. What amount should Nest report as inventory on January 31 under each of the following methods of recording inventory? Perpetual Periodic A.$2,600$5,400 B.$5,400$2,600 C.$2,600$2,600 D.$5,400$5,400

B.$5,400$2,600 Under the LIFO (last-in, first-out) inventory method, goods sold are assumed to be the most recently acquired goods (at their related costs). Therefore, goods remaining (ending inventory) are assumed to be the earliest acquired goods (at their related costs). If the perpetual LIFO inventory method is used, when goods are sold, they are assumed to be the goods acquired just prior to the sale. Thus, Nest's sale of 1,800 units on 1/23 would have consisted of the 1,200 units acquired 1/8 and 600 units (of the 2,000) in beginning inventory. Ending inventory on January 31 would be: 1,400 units of beginning inventory @ $1 each= $1,400 800 units purchased 1/28 @ $5 each= 4,000 2,200 units in ending inventory reported @= $5,400 If the periodic LIFO inventory method is used, ending inventory (and cost of goods sold) are determined only at the end of the period. Therefore, Nest's sale of 1,800 units on 1/23 would have consisted of (by assumption at the end of the period) 800 units acquired on 1/28 and 1,000 units (of the 1,200) acquired on 1/8. Ending inventory on January 31 would be: 200 units of the 1,200 purchased 1/8 @ $3= $ 600 2,000 units (all) of beginning inventory @ $1= 2,000 2,200 units in ending inventory reported @= $2,600

How does the retail inventory method establish the lower-of-cost-or-market valuation for ending inventory? The procedure is applied on a cost basis at the unit level. By excluding net markups from the cost-to-retail ratio. By excluding beginning inventory from the cost-to-retail ratio. By excluding net markdowns from the cost-to-retail ratio.

By excluding net markdowns from the cost-to-retail ratio.

A building suffered uninsured water and related damage. The damaged portion of the building was refurbished with upgraded materials. The cost and related accumulated depreciation of the damaged portion are identifiable. To account for these events, the owner should: Capitalize the cost of refurbishing and record a loss in the current period equal to the carrying amount of the damaged portion of the building. Capitalize the cost of refurbishing by adding the cost to the carrying amount of the building. Record a loss in the current period equal to the cost of refurbishing, and continue to depreciate the original cost of the building. Record a loss in the current period equal to the sum of the cost of refurbishing and the carrying amount of the damaged portion of the building.

Capitalize the cost of refurbishing and record a loss in the current period equal to the carrying amount of the damaged portion of the building. When the portion of an asset that is removed from a larger asset has identifiable costs and accumulated depreciation amounts, those amounts are removed from the books. The difference between these two amounts is the carrying value of the damaged portion of the larger asset. There is no insurance. Therefore, the carrying value of the damaged portion is written off as a loss. The replacement assets are capitalized at cost. The entries are: Portion removed New materials Loss Asset Accumulated depreciation Cash Asset

A building suffered uninsured fire damage. The damaged portion of the building was refurbished with higher quality materials. The cost and related accumulated depreciation of the damaged portion are identifiable. To account for these events, the owner should: Reduce accumulated depreciation equal to the cost of refurbishing. Record a loss in the current period equal to the sum of the cost of refurbishing and the carrying amount of the damaged portion of the building. Capitalize the cost of refurbishing, and record a loss in the current period equal to the carrying amount of the damaged portion of the building. Capitalize the cost of refurbishing by adding the cost to the carrying amount of the building.

Capitalize the cost of refurbishing, and record a loss in the current period equal to the carrying amount of the damaged portion of the building. When the cost and accumulated depreciation of a component or portion of a larger asset is identifiable, and that component or portion is replaced, the replacement is treated as two separate transactions:(1) disposal of the old component (for zero proceeds in this case, due to the fire damage) and(2) purchase of the new component. Thus, a loss equal to the book value of the old component is recognized for (1) and the amount paid to purchase the new component is capitalized as a separate purchase for (2).

On December 31, a company has the following bank accounts and corresponding cash balances: California BankOperating - Summit Ridge($400,000)Operating - Bakersville300,000 Operating - Smithville50,000 Savings500,000 Sedona BankChecking($375,000) How should the company report the above bank account balances in the balance sheet at December 31?

Cash of $450,000 and a liability of $375,000 Correct! The entity can net the overdraft of $400,000 in the California Bank with the positive balances held at the same bank. The net value in the California Bank is $850,000 less $400,000 or $450,000. The $375,000 overdraft in the Sedona Bank is reported as a liability.

Lyon Co. estimated its ending inventory using a method based on the financial statements of prior periods in order to prepare its quarterly interim financial statements. What type of inventory system and method of estimating ending inventory is Lyon using? Inventory system Method of estimating ending inventory A. Perpetual Retail method B. Perpetual Gross profit method C. Periodic Sales method D. Periodic Gross profit method

D. Periodic Gross profit method

A company decided to change its inventory valuation method from FIFO to LIFO in a period of rising prices. What was the result of the change on ending inventory and net income in the year of the change? Ending inventory Net income Increase Increase Increase Decrease Decrease Decrease Decrease Increase

Decrease Decrease Ending inventory would decrease because under LIFO, the latest items purchased (and therefore the most costly) are considered sold, leaving the earliest items purchased (and therefore the least costly) in inventory. This is opposite to the effect under FIFO. The same is true for net income because now, under LIFO, cost of goods sold is increased relative to FIFO because the cost of the latest and most costly items are considered sold first.

A fixed asset with a five-year estimated useful life and no residual value is sold at the end of the second year of its useful life. How would using the sum-of-the-years'-digits method of depreciation, instead of the double declining balance method of depreciation, affect a gain or loss on the sale of the fixed asset? Gain Loss Decrease Decrease Decrease Increase Increase Decrease Increase Increase

Decrease Increase Under SYD, total depreciation through the first two years is [(5 + 4)/(1 + 2 + 3 + 4 + 5)]Cost = (9/15)Cost.Therefore, book value remaining is (6/15)Cost = .4Cost. Depreciation, year one= (2/5)Cost = .4Cost Depreciation, year two= (2/5)(Cost-Depreciation, year one) = (2/5)[Cost-(2/5)Cost] =.4[Cost-.4(Cost)] = .4(.6Cost) = .24 Cost Total depreciation for the two years is therefore .4(Cost) + .24(Cost) = .64(Cost). Book value remaining is (1-.64)Cost = .36 Cost.The asset has a larger book value under SYD after two years. For a given amount of proceeds on disposal, the larger book value under SYD causes any gain on disposal to be smaller than under DDB and any loss greater than under DDB. In other words, the gain decreases and the loss increases, relative to DDB.

Which inventory costing method would a company that wishes to maximize profits in a period of rising prices use? FIFO Dollar-value LIFO. Weighted average. Moving average.

FIFO FIFO assumes the sale of the earliest goods first. With rising prices, the earliest goods reflect the lowest prices. Therefore, cost of goods sold under FIFO is the lowest of the cost flow assumptions. With the lowest cost of goods sold, gross margin and income are the highest among the available cost flow assumptions (LIFO and average being the others).

Bee Co. uses the direct write-off method to account for uncollectible accounts receivable. During an accounting period, Bee's cash collections from customers equal sales adjusted for the addition or deduction of the following amounts: Accounts written off Increase in accounts receivable balance

Deduction Deduction Under the direct write-off method, write-offs are credited directly to accounts receivable (AR). No allowance account is used. Under the terms of the question, accounts receivable increased during the year. Increase in AR = sales − cash collections − write-offs cash collections = sales − increase in AR − write-offs.

A corporation entered into a purchase commitment to buy inventory. At the end of the accounting period, the current market value of the inventory was less than the fixed purchase price, by a material amount. Which of the following accounting treatments is most appropriate? Describe the nature of the contract in a note to the financial statements, recognize a loss in the Income Statement, and recognize a liability for the accrued loss. Describe the nature of the contract and the estimated amount of the loss in a note to the financial statements, but do not recognize a loss in the Income Statement. Describe the nature of the contract in a note to the financial statements, recognize a loss in the Income Statement, and recognize a reduction in inventory equal to the amount of the loss by use of a valuation account. Neither describe the purchase obligation nor recognize a loss on the Income Statement or Balance Sheet.

Describe the nature of the contract in a note to the financial statements, recognize a loss in the Income Statement, and recognize a liability for the accrued loss. The firm has committed to a fixed price but must recognize the loss in the period the decline in price occurred, much like under lower-of-cost-or-market. Inventory is not reduced because the firm has not purchased the inventory under contract. There is no asset to reduce, but the decrease in net assets is accomplished by recording the liability for the portion of the purchase price that has no value.

Red Co. had $3 million in accounts receivable recorded on its books. Red wanted to convert the $3 million in receivables to cash in a more timely manner than waiting the 45 days for payment as indicated on its invoices. Which of the following would alter the timing of Red's cash flows for the $3 million in receivables already recorded on its books? Change the due date of the invoice. Factor the receivables outstanding. Discount the receivables outstanding. Demand payment from customers before the due date.

Factor the receivables outstanding. Factoring is a sale of receivables. This allows Red Co. to sell the receivables and receive cash immediately upon sale.

A company issued a purchase order on December 15, Year 1, for a piece of capital equipment that costs $100,000. The capital equipment was shipped from the vendor on December 31, Year 1, and received by the company on January 5, Year 2. The equipment was installed and placed in service on February 1, Year 2. On what date should the depreciation expense begin? December 15, Year 1 December 31, Year 1 January 5, Year 2 February 1, Year 2

February 1, Year 2 Correct! Depreciation expense should begin on the date that the asset is placed into service and therefore, contributing to the generation of revenues. The depreciation expense should begin on February 1, Year 2.

Alfisol, Inc. offers sales discounts of 2% on all credit sales paid within 15 days. For year 1, gross credit sales totaled $150,000 and 75% of Alfisol's customers took advantage of the discount. Under the net method

For cash receipts after the discount period, discounts not taken must be credited for $750. This answer is correct. Under the net method, sales are initially recorded net of discounts. Payments received after the discount period total $37,500 ($150,000 × 25%), and the amount of discounts forfeited is $750 ($37,500 × 2%). Under the net method, the entry to record these receipts is Cash 37,500 AR 36,750 Discounts not taken 750

Which of the following is not a requirement for an asset to be categorized as a plant asset? Have physical substance. Have a useful life of at least three years. Currently used in operations. Not held for investment purposes.

Have a useful life of at least three years A useful life of at least three years is NOT a requirement for classification of a plant asset. The plant asset must have a useful life extending more than one year beyond the Balance Sheet date.

Plant assets are occasionally acquired by means other than by paying cash. Choose the correct statement about such acquisitions. If equipment is acquired with 100% debt financing, the equipment is capitalized at the sum of all interest and principal payments on the debt. If a building is acquired by issuing an amount of stock that is significant in relation to the amount of stock outstanding before the exchange, the fair value of the building should be used to initially debit the building account. If land is received by a firm as a donation, no amount should be recorded for the land because there is no cost to the firm. If land is acquired as one component of a group of plant assets for a discounted aggregate price, the amount capitalized for the land is its market value.

If a building is acquired by issuing an amount of stock that is significant in relation to the amount of stock outstanding before the exchange, the fair value of the building should be used to initially debit the building account. The more objective or readily determinable value is used for recording the building. If the number of shares is significant in relation to the total shares outstanding, the stock price will be affected by the increase in the shares outstanding resulting from the purchase. The more objective value is the appraised value of the building.

When the allowance method of recognizing bad debt expense is used, the entries at the time of collection of a small account previously written off would

Increase the allowance for doubtful accounts. This answer is correct. The solutions approach is to determine the journal entries necessary to (1) reestablish and (2) collect the account receivable. The entry to reestablish the account would be Accounts receivable xx Allowance for doubtful accounts xx The entry to record collection would be Cash xx Accounts receivable xx The net effect is an increase in a current asset account, cash, and an increase in a contra asset account, allowance for doubtful accounts.

During Year 1, Bay Co. constructed machinery for its own use and for sale to customers. Bank loans financed these assets both during construction and after construction was complete. How much of the interest incurred should be reported as interest expense in the Year 1 Income Statement? Interest incurred for machinery for own use Interest incurred for machinery held for sale All interest incurred All interest incurred All interest incurred Interest incurred after completion Interest incurred after completion Interest incurred after completion Interest incurred after completion All interest incurred

Interest incurred after completion All interest incurred Interest during construction on assets constructed for a firm's own use is capitalized until construction is complete. Thus, only the interest incurred after completion is expensed. Interest is capitalized on the construction of assets for sale only if the assets are large, individual, discrete projects, such as ships or real estate developments. The equipment constructed for sale does not appear to be a discrete item in that sense and, thus, none of the interest is capitalized. It is all expensed.

A company recently moved to a new building. The old building is being actively marketed for sale, and the company expects to complete the sale in four months. Each of the following statements is correct regarding the old building, except: It will be reclassified as an asset held for sale. It will be classified as a current asset. It will no longer be depreciated. It will be valued at historical cost.

It will be valued at historical cost. Only assets used in current operations are included in the category of property, plant and equipment (PPE) on the balance sheet. Assets that are held for sale are reclassified from PPE to 'assets held for sale' and are no longer depreciated. This question is an example of a question framed in the null form. That is, the question wants you to find the exception. This response states the asset will be valued at historical cost—that is false. An asset held for sale is reported at net realizable value.

On June 1, Year 2, Archer, Inc. issued a purchase order to Cotton Co. for a new copier machine. The machine requires one month to produce and is shipped f.o.b. destination on July 1, Year 2, and is received by Archer on July 15, Year 2. Cotton issues a sales invoice dated July 2, Year 2, for the machine. As of what date should Archer record a liability for the machine? June 1, Year 2 July 1, Year 2 July 2, Year 2 July 15, Year 2

July 15, Year 2 Correct! The term f.o.b. destination means that title transfers to the buyer when it arrives at the destination. A liability is recorded when the title transfers.

Generally, which inventory costing method approximates most closely the current cost for each of the following? Cost of goods sold Ending inventory LIFOFIFO LIFOLIFO FIFOFIFO FIFOLIFO

LIFOFIFO LIFO assumes the sale of the most recent purchases first and thus results in cost of goods sold that is the most current value. FIFO assumes the sale of the earliest purchases first (and beginning inventory before any purchases) and thus results in ending inventory that is the most current value. FIFO is sometimes called LISH: last in still here.

If ending inventory for 20x5 is understated because certain items were missed in the count, then: Net income for 20x5 will be overstated. CGS for 20x5 will be understated. Net income for 20x5 will be understated, but net income for 20x6 will be unaffected. Net income for 20x5 will be understated and CGS for 20x6 will be understated.

Net income for 20x5 will be understated and CGS for 20x6 will be understated Use the equation BI + PUR = EI + CGS. When EI is understated, CGS must be overstated to maintain the equation. Net income, therefore, is understated (20x5). Then next year, BI is also understated because BI for 20x6 is EI for 20x5. Using the equation, if BI is understated, CGS is also understated to maintain the equation.

The original cost of an inventory item is above the replacement cost. The inventory item's replacement cost is above the net realizable value. Under the lower of cost or market method, the inventory item should be valued at Original cost. Replacement cost. Net realizable value. Net realizable value LESS normal profit margin.

Net realizable value. Inventory must be carried at lower of cost (such as LIFO or market. Market is replacement cost subject to a ceiling and floor. The ceiling for replacement cost is net realizable value (selling price less cost to complete) and the floor is net realizable value less normal profit margin. Use simple numbers to help solve this abstract question. In this question original cost (assume = 100) is greater than market ((replacement cost) assume = 80). Market (80) is greater than net realizable value (assume = 70). Market is subject to a ceiling of net realizable value (70). In this case the inventory would be valued at net realizable value.

A depreciable asset has an estimated 15% salvage value. Under which of the following methods, properly applied, would the accumulated depreciation equal the original cost at the end of the asset's estimated useful life? Straight-line Double-declining balance Yes Yes Yes No No Yes No No

No No Salvage value is the portion of the asset's cost not subject to depreciation. Total depreciation, under any method, is limited to depreciable cost (cost less salvage value). The declining balance methods do not subtract salvage when computing depreciation. Care must be taken to avoid depreciating an asset beyond salvage value.

Choose the correct inclusions to the cost-to-retail ratio computation under the dollar-value LIFO retail method. Beginning Inventory Net Markdowns Yes Yes Yes No No Yes No No

No Yes DV LIFO retail uses the FIFO (not LCM) cost-to-retail ratio. Under LIFO, a layer added during a period should reflect only the cost and retail amounts pertaining to that period. Thus, beginning inventory amounts are not used in calculating the ratio. Also, because LIFO may contain inventory layers for several preceding periods, excluding net markdowns is not an effective way to accomplish the LCM valuation objective. Thus, net markdowns are included in the cost to retail computation.

Many years after constructing a plant asset, management spent a significant sum on the asset. Which of the following types of expenditures should be capitalized in this instance: (1) an expenditure for routine maintenance that increases the useful life compared with deferring the maintenance, (2) an expenditure that increases the useful life of the asset compared with the original estimate assuming normal maintenance at the required intervals, (3) an expenditure that increases the utility of the asset. (1) (2) (3) Yes Yes Yes No Yes Yes No No Yes No Yes No

No Yes Yes Post-acquisition expenditures, which increase the useful life (assuming normal maintenance) or the utility (usefulness or productivity) of the asset, are capitalized. Such expenditures provide value for more than one year. The original useful life of an asset assumes regular maintenance. Therefore, regular maintenance does not increase the intended useful life of the asset.

When the allowance method of recognizing uncollectible accounts is used, how would the collection of an account previously written off affect accounts receivable and the allowance for uncollectible accounts? Accounts receivable Allowance for uncollectible accounts

No effect Increase

During the year, Hauser Co. wrote off a customer's account receivable. Hauser used the allowance method for uncollectible accounts. What impact would the write-off have on net income and total assets? Net income Total assets

No effect No effect Under the allowance method for uncollectible accounts there is no impact on the balance sheet or net income when the receivable is written off. The estimated uncollectible is recognized at the time of the sale; therefore, when the account is written off, the allowance and the accounts receivable are both reduced resulting in no effect on the income statement or balance sheet.

On January 1, 20X5, Brecon Co. installed cabinets to display its merchandise in customers' stores. Brecon expects to use these cabinets for five years. Brecon's 20X5 multi-step Income Statement should include: One-fifth of the cabinet costs in cost of goods sold. One-fifth of the cabinet costs in selling, general, and administrative expenses. All of the cabinet costs in cost of goods sold. All of the cabinet costs in selling, general, and administrative expenses.

One-fifth of the cabinet costs in selling, general, and administrative expenses. With a five year life, 1/5 of the cost of the cabinets is expensed as depreciation. The cabinets are not involved in the manufacturing of the goods. Rather, they are used to help sell the merchandise. Thus, the depreciation is not included in cost of goods sold; rather, it is included in selling, general, and administrative expenses.

Newt Co. sold a warehouse and used the proceeds to acquire a new warehouse. The excess of the proceeds over the carrying amount of the warehouse sold should be reported as a(an): Reduction of the cost of the new warehouse. Gain from discontinued operations, net of income taxes. Part of continuing operations. Extraordinary gain, net of taxes.

Part of continuing operations. The gain or loss on the sale of an asset is part of continuing operations as it is expected that a company will sell existing assets from time to time as the assets are replaced.

The retail inventory method includes which of the following in the calculation of both cost and retail amounts of goods available for sale? Purchase returns. Sales returns. Net markups. Freight in.

Purchase returns The retail method measures beginning inventory and net purchases at both cost and retail. It then applies the average relationship between cost and retail (based on beginning inventory and purchases) to ending inventory at retail to determine ending inventory at cost. Purchase returns reduce net purchases at both cost and retail because returns represent amounts included in gross purchases that are not available for sale.

The replacement cost of an inventory item is below the net realizable value and above the net realizable value less a normal profit margin. The inventory item's original cost is above the net realizable value. Under the lower of cost or market method, the inventory item should be valued at Original cost. Replacement cost. Net realizable value. Net realizable value less normal profit margin.

Replacement cost. The easiest way to answer a question like this is to make up simple numbers. The following simple numbers were made up to fit the abstract information in the question. Lower of cost or market states you record the inventory at the lower of original cost or market value (replacement cost) within the range of a ceiling and a floor. The numbers below show that replacement cost is lower than original cost and within the floor and ceiling. Replacement cost is the correct answer. Original cost $10 Net realizable value 9 Replacement cost 8 NRV less normal PM 7

On January 1, Year 1, Crater, Inc. purchased equipment having an estimated salvage value equal to 20% of its original cost at the end of a 10-year life. The equipment was sold December 31, Year 5, for 50% of its original cost. If the equipment's disposition resulted in a reported loss, which of the following depreciation methods did Crater use? Double declining balance. Sum-of-the-years'-digits. Straight-line. Composite.

Straight-line. The asset was sold when 1/2 of its useful life was expired. (The asset was used 5 years and had an original useful life of 10 years.) If an asset is sold at a loss, then the book value at the date of sale exceeds the proceeds from sale by the amount of the loss. Let C = original cost, and BV = book value at date of sale. Then BV-proceeds = loss Proceeds = .50C according to the question data. Thus, BV-.50C = loss. Thus, BV must exceed 50% of the original cost because BV-.50C is a positive number. The only method from among those listed in the answer alternatives that leaves a BV greater than 50% of original cost after 50% of the useful life has expired is the SL method. The book value after the fifth year under SL is C-(C-.2C)(5/10) = .6C. DDB's book value after five years is much less than 50% of original cost because it is an accelerated method. The same holds for SYD. And under composite methods of depreciation, individual assets do not have a separately recorded book value. When sold, accumulated depreciation is debited for the difference between original cost and proceeds. No gain or loss is recognized. Thus, the composite method could not apply in this question.

Lano Corp.'s forestland was condemned for use as a national park. Compensation for the condemnation exceeded the forestland's carrying amount. Lano purchased similar, but larger, replacement forestland for an amount greater than the condemnation award. As a result of the condemnation and replacement, what is the net effect on the carrying amount of the forestland reported in Lano's Balance Sheet? The amount is increased by the excess of the replacement forestland's cost over the condemned forestland's carrying amount. The amount is increased by the excess of the replacement forestland's cost over the condemnation award. The amount is increased by the excess of the condemnation award over the condemned forestland's carrying amount. No effect, because the condemned forestland's carrying amount is used as the replacement forestland's carrying amount.

The amount is increased by the excess of the replacement forestland's cost over the condemned forestland's carrying amount. The two transactions are not related. The land account is decreased by the book value of the land condemned and increased by the cost of the land purchased. The relative magnitudes of the book values are shown below: award > book value of condemned landcost of new land > award Therefore: cost of new land > book value of condemned land Thus, the land is increased by the net amount: cost of new land-book value of old land

Losses on purchase commitments are recorded at the end of the current year when: The current cost of the inventory is less than the inventory cost in the purchase contract. The purchase contract is irrevocable. The contractual cost of the inventory in an irrevocable purchase contract exceeds the current cost. The buyer purchased a quantity of inventory that was not sufficient to avoid a LIFO liquidation.

The contractual cost of the inventory in an irrevocable purchase contract exceeds the current cost. Both qualities are required for a loss to be recognized. The firm must honor a contract in a later period by paying more than current cost and, thus, is in a loss position at the end of the current year.

Herc Co.'s inventory on December 31, 2005 was $1,500,000, based on a physical count priced at cost, and before any necessary adjustment for the following: Merchandise costing $90,000, shipped FOB shipping point from a vendor on December 30, 2005, was received and recorded on January 5, 2006. Goods in the shipping area were excluded from inventory although shipment was not made until January 4, 2006. The goods, billed to the customer FOB shipping point on December 30, 2005, had a cost of $120,000. What amount should Herc report as inventory in its December 31, 2005, balance sheet? $1,500,000 $1,590,000 $1,620,000 $1,710,000

The correct ending inventory balance is $1,710,000 ($1,500,000 + $90,000 + $120,000). The $90,000 of merchandise is included because it was shipped before year-end and the title was transferred to Herc at the shipping point (before year-end). The $120,000 also is included because the goods have not been shipped. The FOB designation is irrelevant because the goods have not yet reached a common carrier.

At the end of 20x4, a firm recognized a loss on a contractual commitment to purchase inventory for $60,000. The value of the inventory at the end of 20x4 is $52,000. When the inventory was actually purchased in 20x5, its value had risen to $62,000. Choose the correct statement concerning reporting in 20x5. A $10,000 gain is recognized. The inventory is recorded at $60,000. The inventory is recorded at $52,000. There is no additional loss or gain recognized.

The inventory is recorded at $60,000. The maximum recorded value of the inventory is $60,000, which is the contractual amount and, also, the cost. If the firm can sell the inventory for more than $60,000, then gross margin will be recognized. The value of the inventory more than fully recovered, but gains are limited to the amount of previously recognized losses, which in this case, is $8,000.

When the FIFO inventory method is used during periods of rising prices, a perpetual inventory system results in an ending inventory cost that is The same as in a periodic inventory system. Higher than in a periodic inventory system. Lower than in a periodic inventory system. Higher or lower than in a periodic inventory system, depending on whether physical quantities have increased or decreased.

The same as in a periodic inventory system. FIFO produces the same results for periodic and perpetual systems. FIFO always assumes the sale of the earliest goods acquired. Therefore, unlike LIFO periodic, goods can never be assumed sold before they are acquired. Cost of goods sold and ending inventory are the same under FIFO for both a periodic and a perpetual system.

On October 31, Dingo, Inc. had cash accounts at three different banks. One account balance is segregated solely for a November 15 payment into a bond sinking fund. A second account, used for branch operations, is overdrawn. The third account, used for regular corporate operations, has a positive balance. How should these accounts be reported in Dingo's October 31 classified balance sheet?

The segregated account should be reported as a noncurrent asset, the regular account should be reported as a current asset, and the overdraft should be reported as a current liability. The accounts are with different banks. Thus, the accounts cannot be offset against one another. The overdraft is a liability because the bank honored a check or withdrawal causing the account to be negative. The firm owes the bank this amount. The regular corporate account is part of the cash account, a current asset. The segregated account is a long-term investment. The cash in this asset is set aside for a specific purpose. There is no intent to use the cash for ordinary operating purposes.

What factor must be present to use the units of production (activity) method of depreciation? Total units to be produced can be estimated. Production is constant over the life of the asset. Repair costs increase with use. Obsolescence is expected.

Total units to be produced can be estimated. Without an estimate for total units to be produced, depreciation could not be computed. Annual depreciation under this method is: [(Cost-salvage value)/(Total estimated production)](units produced each year). The quantity in square brackets is the rate of depreciation per unit.

Bren Co.'s beginning inventory at January 1, 2005 was understated by $26,000, and its ending inventory was overstated by $52,000. As a result, Bren's cost of goods sold for 2005 was: Understated by $26,000. Overstated by $26,000. Understated by $78,000. Overstated by $78,000.

Understated by $78,000 The effect of the beginning-inventory error is to understate cost of goods sold $26,000. The effect of the ending-inventory error is to understate cost of goods sold $52,000. The total effect then is to understate cost of goods sold $78,000. These effects are analyzed by using the equation: Beginning inventory + Purchases-Ending inventory = Cost of goods sold For example, if beginning inventory is understated, then the right hand side of the equation (cost of goods sold) must also be understated by the same amount.

Kahn Co., in applying the lower of cost or market method, reports its inventory at replacement cost. Which of the following statements are correct? The original cost is greater than replacement cost The net realizable value, less a normal profit margin, is greater than replacement cost YesYes YesNo NoYes NoNo

Yes No Under LCM, the market value of inventory is the middle of three figures (in amount): replacement cost net realizable value net realizable value less normal profit margin. If the middle figure (market) is less than cost, then the inventory is reported at market. The inventory in this question is reported at replacement cost, which means that replacement cost is market value and replacement cost is less than cost. Also, replacement cost is the middle of the three figures (or tied with one of the other two). Net realizable value less normal profit margin could not exceed replacement cost because that would imply that replacement cost is the lowest of the three figures, which contradicts the fact that replacement cost is market value. Therefore, in terms of the question,(1) original cost is greater than replacement cost, and(2) net realizable value less normal profit margin is not greater than replacement cost.

During periods of inflation, a perpetual inventory system would result in the same dollar amount of ending inventory as a periodic inventory system under which of the following inventory valuation methods? FIFO LIFO Yes No Yes Yes No Yes No No

Yes No Under a perpetual inventory system, the cost of goods sold (COGS) is determined at the time of each sale. In a perpetual FIFO inventory system, the cost of each sale (COGS) would be based on the cost of the earliest acquired goods on hand at the time of the sales. The cost of the most recently acquired goods would remain in ending inventory. In a perpetual LIFO inventory system, the cost of each sale (COGS) would be based on the cost of goods acquired just prior to the sale. The cost of the earlier acquired goods would remain in inventory. Under a periodic inventory system, the costs of goods sold (COGS) and ending inventory are determined only at the end of the period. In a periodic FIFO inventory system, the cost of sales for the period (COGS) would be based on the cost of the earliest acquired goods available during the period. The cost of the most recently acquired goods would remain in ending inventory. In a periodic LIFO inventory system, the cost of sales for the period (COGS) would be based on the last goods acquired during the period. The cost of the earliest acquired goods would remain in ending inventory. The above descriptions can be summarized as follows for determination of COGS: Inventory System/MethodCost of goods sold determined using FIFOCost of goods sold determined using LIFOPerpetualEarliest goods acquiredLatest goods acquired prior to each salePeriodicEarliest goods acquiredLatest goods acquired during the period Since cost of goods sold for the period would be the same under both perpetual FIFO and periodic FIFO, ending inventory would be the same under both perpetual FIFO and periodic FIFO. Cost of goods sold (and ending inventory) would not be the same under perpetual LIFO as under periodic LIFO because the perpetual system recognizes cost of goods sold based on the cost of goods acquired just prior to each sale, whereas the periodic system recognizes cost of goods sold based on cost of goods acquired prior to the end of each period.

Choose the correct accounting by the creditor for a loan impairment. Column (1): recognize a loss or expense upon recognizing the impairment. Column (2): rate of interest to use in computing the revised book value of the receivable after the impairment. 1 2 Yes Original effective rate Yes New implied effective rate No Original effective rate No New implied effective rate

Yes Original effective rate You Answered Correctly! A loan impairment is recorded by reducing the net book value of the receivable to the present value of probable future cash inflows, discounted at the original rate in the receivable. The original rate is used because the loan continues to exist. The loss to the firm is measured at the rate existing when the original loan was created. The difference between the book value and present value, at the date of recognizing the impairment, is recorded as an expense or loss. There is no reason to report overstated assets.

Derby Co. incurred costs to modify its building and to rearrange its production line. As a result, an overall reduction in production costs is expected. However, the modifications did not increase the building's market value, and the rearrangement did not extend the production line's life. Should the building modification costs and the production line rearrangement costs be capitalized? Building modification costs Production line rearrangement costs Yes No Yes Yes No No No Yes

Yes Yes The criterion for capitalizing post-acquisition costs is not whether the market value of the overall asset is increased. Rather, the criteria are (1) increase in useful life or (2) increase in productivity or efficiency including cost reduction. An overall reduction in production costs meets the second criterion. Therefore, both costs are capitalized rather than immediately expensed.

A creditor's note receivable has a carrying value of $60,000 at the end of Year 1. Based on information about the debtor, the creditor believes the note is impaired and establishes the new carrying value of the note to be $25,000 at the end of Year 1. During Years 2 and 3, the debtor pays $14,000 on the note each year (total payments, $28,000). For Year 3, under which method of the two indicated is interest revenue recognized? Interest Method Cost Recovery Method Yes Yes No No Yes No No Yes

Yes Yes The interest method recognizes interest revenue each year until the note is collected because the note was written down to present value when the impairment was recorded. The estimated future cash flows to be received include interest, which is recognized over the remaining term of the note. The cost recovery method recognizes interest revenue only after cash equal to the new carrying value is collected. During Year 3, total collections surpassed the $25,000 new carrying value. $3,000 of interest revenue is recognized under this method in Year 3 ($28,000 − $25,000).

Net income is understated if, in the first year, estimated salvage value is excluded from the depreciation computation when using the Straight-line method Production or use method Yes No Yes Yes No No No Yes

Yes Yes When salvage value is excluded from the computation of depreciation, excessive depreciation is recognized each year under BOTH methods. Therefore, income is understated for both methods. Annual depreciation under straight-line is: (1/n)(cost-salvage) where n is the number of years in the useful life. Annual depreciation under the production method is:(current year production/tot.est.production)(cost-salvage) In both cases, if salvage value is excluded from the computation, depreciation is overstated because cost, rather than depreciable cost, is used as the basis for depreciation.

Two approaches are available for applying interest rates to average accumulated expenditures for the purpose of capitalizing interest. These approaches are called the specific method and the weighted average method. In some cases, these approaches yield the same results. Two situations may be encountered in practice for a specific period: (1) average accumulated expenditures exceed total interest bearing debt (principal) and (2) the interest rates on all interest bearing debt instruments are the same. Which situation yields the same results for the two approaches? only (1). only (2). both (1) and (2). neither (1) nor (2).

both (1) and (2). When average accumulated expenditures exceeds interest bearing debt, all interest for the period is capitalized because all debt could have been avoided if the construction had not taken place. Also, if the interest rates on all debt are the same, then the two approaches yield the same results because, ultimately, only one interest rate is applied to average accumulated expenditures for computing capitalized interest.

Debt is frequently incurred when plant assets are acquired. For example, debt may be incurred on the purchase of plant assets. Debt may also be incurred during the construction of plant assets. How is the interest in these two cases treated for financial reporting? Debt for purchase Debt during construction expense capitalize expense expense capitalize capitalize capitalize expense

expense capitalize Interest on debt incurred when purchasing a plant asset, is incurred after the asset has reached its intended condition and location. Therefore, it is expensed as incurred. Debt incurred during the construction of plant assets is considered avoidable and also incurred before the asset has reached its intended condition and location. Therefore, it is capitalized to the asset in the same way material, labor, and overhead are capitalized. The interest is expensed as part of depreciation during the service life of the asset.


Related study sets

Insurance Exam - May 2024 - Georgia

View Set

UNIT 22 series 65 trading securities

View Set

321 final - ch38: assess/manage of pts w rheumatic disorders

View Set

Ellis Advanced Grammar, Ellis Advanced Vocabulary

View Set